MPRE Multiple Choice Q's

Ace your homework & exams now with Quizwiz!

Pablo, a lawyer who used to work at the U.S. Department of Justice, now works at a private law firm. Which of the following rules does NOT apply to potential conflicts between Pablos duties to the U.S. government and his duties to any current clients? A. 1.7 B. 1.9(a) C. 1.9(c) D. 1.11

B. 1.9(a)

Membership in a state bar association is required for a person who is being licensed to practice law: A. By some states. B. By no state. C. By every state, unless the person is a member of a bar association in another state. D. By every state, even if the person is member of a bar association in another state.

A. By some states.

Joseph is an immigrant who was recently denied asylum by the administrative appeals body that handles immigration cases. He hires Cyrus to appeal the decision to the US: Court of Appeals, Joseph knows that there is a $500 fling fee that he will have to pay. The appeal and fee are due tomorrow. Cyrus called Joseph and left a message, but Joseph did not call him back. He knows from prior conversations, however, that Joseph wants to appeal the decision. If he misses the filing deadline, Joseph will be barred from appealing. May Cyrus go ahead and file the appeal, paying the filing fee, with the expectation that Joseph would reimburse him? A. Yes, because lawyers are permitted to advance court costs. B. Yes, because $500 is not excessive. C. No, because lawyers are not permitted to provide financial assistance to clients. D. No, because Joseph did not exercise reasonable diligence in communicating with Cyrus before advancing the fee.

A. Yes, because lawyers are permitted to advance court costs.

Attorney Aria represents Jaya, who is a defendant in a securities fraud prosecution. One day, Aria is interviewing Jaya about the allegations that she engaged in insider trading. laya, who is married, brings a man named Josh to the interview, and she reveals to Aria that she is having an affair with him. She further reveals that the Josh provided her with some inside information. Josh is not a defendant in the case and is not represented by Aria. Jaya's statements about Josh are: A. Confidential and privileged. B. Confidential, but not privileged. C. Not confidential, but privileged. D. Neither confidential nor privileged.

B. Confidential, but not privileged.

Last year, attorney Mira represented Sally, the owner of a small pizza restaurant, in a suit against Romes Own, Inc., a supplier of shredded cheese, based on breach of contract. The parties settled the claim for $15,000. Mira and Sally remained friendly after the matter was settled, and have gone to the movies and to dinner a few times in the last year. Earlier today, Sally's next door neighbor, Jose, came in to Mira's office for a consultation and asked Mira to represent him in a civil suit against Sally. Jose alleges that Sally's pet macaw bit him, causing an infection and permanent scarring. Jose wants to get a court order against Sally requiring that the bird be euthanized pursuant to a state law. Sally already paid for his medical care. Jose does not want further damages. Must Mira obtain Sally's informed consent before she can agree to represent Jose? A. Yes, because Jose's interests are directly adverse to Sally's. B. Yes, because there is a significant risk that Mira's representation of Jose will be materially limited by her loyalty to Sally. C. No, because Jose's lawsuit is not substantially related to the breach of contract lawsuit against Rome's Own, Inc. D. No, because Jose's interests are not materially adverse to Sally's.

C. No, because Jose's lawsuit is not substantially related to the breach of contract lawsuit against Rome's Own, Inc.

A law firm has offices in three major U.S. cities. Once a year, lawyers from all three offices convene for a firm retreat. One of the partners in the firm, Sandler, represents Nigel, a celebrity chef, in several lawsuits (filed both by and against Nigel). Sandler prepares a complex counseling exercise for the retreat, based on his relationship with Nigel, called "Dealing with a Difficult Client" Sandler does not change Nigel's name or any of the facts in any of the training materials. The materials provide detailed descriptions of cases in which Nigel was a party, and included stories about difficult interactions between the chef and his lawyers. The materials are distributed in loose leaf notebooks clearly marked "confidential training materials." Nigel learns about the training program from Tommy, one of the firm's junior lawyers, whom Nigel is dating casually (unbeknownst to Sandler). Nigel is furious. Is Sandler subject to discipline for violation of Rule 1.6? A. Yes. because that information was obtained in the course of representation, and the disclosure was not impliedly authorized to carry out the representation. B. Yes, because Sandler did not obtain Nigel's consent to the use of this information in the training. C. Yes, because Sandler disclosed confidences to lawyers who work in offices of the law firm that are located in other cities. D. No.

D. No.

Sophia retained Drake as her attorney to sue the city where she lived after she stepped into a deep hole in the sidewalk, fell, and broke her leg. She incurred $4,000 in medical bills and suffered a great deal of pain. State law provides that a city is not liable in suits for personal injury unless the suit is filed within one year after the injury occurs. Drake never filed the suit. Six months after the one-year period was up, Sophia sued Drake for malpractice, based on his failure to file the lawsuit. In fact, Drake called Sophia five days before the one-year period was up and told her that he was ready to file, and Sophia told him not to go ahead with the case. When he asked why not, she said, "It is because I am a finalist for a job as the city's commissioner of parks, and I probably won't get the job if I sue the city. I'm telling you this in the strictest confidence. If I don't get the job, I don't want anyone, including my husband, to know that I applied for this job and didn't get it." Sophia was not selected for the job. She decided to sue Drake because she needed compensation for her injuries. Drake asserted, in his answer to Sophias malpractice complaint, that Sophia told him not to file her suit because she was hoping to get a job as commissioner of parks. He limited the disclosure to the minimum necessary to defend himself against this spurious malpractice claim. Is Drake subject to discipline? A. No, because he revealed no more than necessary to defend himself. B. No, because Sophia provided the information to Drake before she asked for strict confidence about her intention to sue the city. C. Yes, because lawyers must not reveal confidential client information, especially in cases in which the client has expressly told the lawyer that a secret must be kept in strict confidence. D. Yes, because Drakes revelation of information adverse to Sophia is a breach of fiduciary duty.

A. No, because he revealed no more than necessary to defend himself.

Attorney Kristin is a state prosecutor, and she is prosecuting Oliver Burson for the murder of his ex-wife. Before the trial begins, a reporter asks to interview Kristin for a television report. Which of the following statements would Kristin be well-advised NOT to make during the interview? A. "We are investigating the murder of Oliver Burson's ex-wife, Nina Richards" B. "Oliver Burson, who murdered his ex-wife, will be prosecuted to the fullest extent of the law.?" C. "Oliver Burson lives at 530 Orange Street." D. "Oliver Burson was arrested on January 22nd."

B. "Oliver Burson, who murdered his ex-wife, will be prosecuted to the fullest extent of the law.?"

Valladia, Inc., is a small, closely held corporation. Valladia and its president, Alan, are defendants in a civil action brought by a state attorney general who accuses both of them of fraud, based on the same facts and the same law. Both of them desire to be represented by Martha, an attorney. Martha will bill each defendant for services provided on behalf of that defendant. Martha reasonably believes that she could provide competent and diligent representation to both of them. Martha advises the board of directors of Valladia of all the foreseeable risks of the corporation being represented by the same attorney who is representing Alan, and she advises Alan of all of the foreseeable risks of being represented by the lawyer who is representing the board. Valladias board of directors votes to approve the representation, and Alan approves as well. Martha writes a letter to the corporation and to Alan noting that both have given their informed consent to her representation of both parties. May Martha represent both of them? A. Yes, because the corporation gave informed consent to the joint representation. B. Yes, because both defendants gave informed consent to the joint representation. C. No, because a single lawyer may not represent two co-defendants in civil litigation. D. No, because even though conflicts are not foreseeable at present, they could arise in the future.

B. Yes, because both defendants gave informed consent to the joint representation.

Herman was mugged in a dark alley, and his wallet, which contained $200 in $20 bills, was stolen. Moments after the assailant fled with his money, Herman spotted a police officer and reported the crime. Herman described the assailant as a male who had dark hair, was of medium height, and wore a dark colored shirt and white sneakers. He said the assailant had showed him a knife. The officer alerted other officers in the vicinity by radio. An officer arrested Juan three blocks away. Juan has dark hair and is of medium height. He was wearing a dark blue shirt and white sneakers. When searched, he was found to have $454 in cash in his pocket, including eight twenty dollar bills. He was not in possession of a knife. When arrested, Juan acted suspiciously and refused to answer any questions. He has not confessed. Herman's wallet was never found, and no knife was found in the vicinity. The case was assigned to Marcella, a prosecutor. Marcella investigates but discovers no additional facts. She believes that Juan is guilty, though she doubts that a jury would find him guilty beyond a reasonable doubt. In fact, she is uncertain whether the charge is even supported by probable cause. Is it proper for Marcella to charge Juan with robbery? A. Yes, because she believes that Juan is guilty. B. Yes, because she does not know that the charge is not supported by probable cause. C. No, because she doubts that a jury would convict him. D. No, if she thinks that he is not guilty beyond a reasonable doubt.

B. Yes, because she does not know that the charge is not supported by probable cause.

Julie works for a federal government agency. Agency regulations prohibit lawyers employed by the agency from representing individual clients. Julie wants to fulfill the aspirational requirements of pro bono service as stated in Rule 6.1. May she fulfill the Rule's aspirational requirements without quitting her job? A. Yes, by contributing a significant amount of money to the legal assistance program run by her state bar association. B. Yes, by serving on a bar association committee that makes recommendations for law reform. C. No, because in order to fulfill the aspirational requirements of Rule 6.1, lawyers must represent low-income clients without charging fees. D. No, because she cannot satisfy the aspirational requirements of Rule 6.1 without violating the agency's prohibition on representing individual clients.

B. Yes, by serving on a bar association committee that makes recommendations for law reform.

Use the facts from the previous question. Must Mira obtain Jose's informed consent before she can agree to represent Jose? A. Yes, because Jose's interests are directly adverse to Sally's. B. Yes, because there is a significant risk that Miras representation of Jose will be materially limited by her loyalty to Sally. C. No, because Jose's lawsuit is not substantially related to the breach of contract lawsuit against Rome's Own, Inc. D. No, because Joses interests are not materially adverse to Sallys.

B. Yes, because there is a significant risk that Miras representation of Jose will be materially limited by her loyalty to Sally.

Nathan is a partner in the law firm of Pace and Gillespie, which represents Bostwick Corp., which is not publicly traded. The corporation manufactures boots. Nathan is a friend of David Bostwick, the chairman of his client's board of directors, who owns 12 percent of the stock of the company. Nathan himself owns 3 percent of the stock, which he acquired long before Pace and Gillespie began representing Bostwick. Whenever there is a vacancy on the board, the board selects a replacement director. No directors are elected by the shareholders. On David's recommendation, the board takes a formal vote and offers Nathan the opportunity to join the board. Nathan would like to accept, and neither he nor anyone on the board knows of any actual or apparent conflict of interest. Nathan is not willing to divest himself of his stock in the company. May Nathan accept the offer to join the board? A. Yes, because there is no law or ethical rule barring his joining the board. B. Yes, because although Nathan is a friend of David, the board took a formal vote to appoint him. C. No, because he is a friend of David and therefore there is at least the appearance of a conflict of interest, regardless of the opinion of Nathan or board members. D. No, because Nathan refuses to divest himself of stock in the company.

A. Yes, because there is no law or ethical rule barring his joining the board.

Attorney Sebastian graduates from law school and starts his own criminal defense practice. He establishes an LLC for his practice and sets up a bank account for the office expenses. He also has a personal bank account. He decides to charge the relatively low fee of $200 per hour for his work. Within the first month of opening his practice, five different clients retain him in connection with their criminal matters. Pursuant to the retainer agreements, each client gives Sebastian a check for $4,000. Sebastian will earn $200 for each hour that he works. After 20 hours, he will bill each client for additional sums. At a minimum, how many additional bank accounts must Sebastian open? A. 0 B. 1 C. 4 D. 5

B. 1

Attorney Moira represents defendant Oleg in a murder trial. The police discovered the victim's body in a ditch and found Olegs fingerprints on a kitchen knife that was under the body. Before the trial commences, Oleg tells Moira that he did not murder the victim, but he knows that his son did, because his son confessed to him. He tells her that under no circumstances will he turn his son in, and he does not want his son to know that he told Moira of his son's confession. He refuses to testify at his trial. What is Moiras best course of action? A. Reveal the information to the prosecutor to avoid a miscarriage of justice but remain as Oleg's attorney if he wants her to do so. B. Reveal the information to the prosecutor to avoid a miscarriage of justice but move to withdraw from representing Oleg. C. Tell Oleg's son that she has learned of his confession to his father and urge Oleg's son to retain her as his lawyer so that she will have an obligation not to reveal the information. D. Do not reveal the information and do not tell Oleg's son that she is aware of his confession to his father.

D. Do not reveal the information and do not tell Oleg's son that she is aware of his confession to his father.

Attorney Meghan represents Walter, a used-car salesman who has been criminally charged with driving while intoxicated. Before Walter's trial, a friend of Meghan's, Laurel, asks Meghan to represent her in her breach of contract suit against Walter and his company. If Meghan were to sue Walter on behalf of Laurel, another lawyer would defend Walter in that matter. Meghan has formed a close bond with Walter as a result of her work on his criminal case, and she reasonably believes that she could not represent Laurel very vigorously, because of her friendship with Walter. She explains her concern to Laurel, and Laurel states that she understands the risks and agrees to the representation anyway. She confirms her understanding in writing. Walter also gives informed consent in writing. May Meghan represent Laurel? A. Yes, because the representation is not prohibited by law. B. Yes, because Walter's case is criminal and Laurel's case is civil. C. No, because the representation would involve the assertion of a claim by one client against another client represented by the lawyer in the same litigation. D. No, because Meghan does not reasonably believe that she can competently and diligently represent Walter and Laurel, even though both clients gave informed consent, confirmed in writing.

D. No, because Meghan does not reasonably believe that she can competently and diligently represent Walter and Laurel, even though both clients gave informed consent, confirmed in writing.

Chloe and Matilda are partners in a small law firm. Chloe has represented Speisler Motors, an auto manufacturer, for years. She represents Speisler in all litigation in which it is a party. At present she is defending the company in a lawsuit brought by Tom, a driver of a speisler Car who was blinded when an airbag spontaneously exploded while he was driving. When the bag exploded, Tom's car hit Anton, a pedestrian, who died some days after the accident. The executor of Anton's estate has asked Matilda to represent the estate in a lawsuit against Tom and Speisler Motors. Matilda believes that she would be able to provide competent and diligent representation to Anton's estate, and Chloe believes that she would be able to provide competent and diligent representation to Speisler Motors. They have agreed not to talk with each other or to share documents relating to the matter. Speisler, Tom, and Anton's executor all give verbal consent to the representation after full disclosure of all material risks. Is it proper for Matilda to accept representation of Anton's estate? A. Yes, because Matilda could reasonably believe that she would be able to provide competent and diligent representation to Anton's estate, and Chloe could reasonably believe that she would be able to provide competent and diligent representation to Speisler Motors. B. Yes, because Matilda and Chloe have agreed not to talk with each other or to share documents relating to the matter. C. No, because although the conflict is consentable, neither Matilda nor Chloe confirmed her client's informed consent in writing. D. No, because there is a conflict of interest and the conflict is not consentable.

D. No, because there is a conflict of interest and the conflict is not consentable.

Attorney Darla represents the plaintiffs in a class action lawsuit against Nutrisnax, a granola bar company, alleging false advertising. The plaintiffs allege that Nutrisnax falsely advertised its granola bars as a healthy alternative to candy bars when, in fact, the granola bars contained just as much fat, sugar, and sodium as the average candy bar. The class includes anyone who purchased a Nutrisnax granola bar in the last three years. Last week, a potential client named Samuel came into Darla's office and asked her to represent him in his claim for child support against his former husband, Franklin. Darla learned that Franklin purchased hundreds of Nutrisnax granola bars during the relevant period, making him one of the unnamed members of the class in the granola bar suit. Must Darla obtain Franklins informed consent before agreeing to represent Samuel? A. Yes, because Samuel's interests are directly adverse to Franklin's. B. Yes, because Darla cannot reasonably believe that she can competently and diligently represent both Franklin and Samuel. C. No, because the child support claim is unrelated to the class action lawsuit. D. No, because unnamed members of a class are ordinarily not considered to be clients of the lawyer for conflicts purposes.

D. No, because unnamed members of a class are ordinarily not considered to be clients of the lawyer for conflicts purposes.

Which of these bodies sometimes exercise functions that are delegated or authorized by governmental institutions? A. State bar associations. B. The Section of Legal Education and Admissions to the Bar of the American Bar Association. C. Both A and B. D. The American Law Institute.

C. Both A and B

Which one of the following communications would be protected by the attorney-client privilege? A. A prospective client tells an attorney the facts of her case, but the attorney decides not to take the case. B. An attorney and client meet in a crowded restaurant to discuss the client's case. They do not know anyone sitting around them, but they are sitting near enough to other diners that others could hear their conversation. In fact, no one hears the conversation. C. A lawyer represents a client in a dispute about a real estate contract. The client gives the attorney a copy of the contract at issue. The contract includes statements about the obligations of both parties and statements about their reasons for entering into the contract. D. A lawyer and a client go out to celebrate after obtaining a favorable settlement. While they are out to dinner, they talk at length about the personalities of the opposing party in the case, that person's lawyer, and the judge.

A. A prospective client tells an attorney the facts of her case, but the attorney decides not to take the case.

Winthrop, a judge, and his brother Leonard, a lawyer, have both been members, for many years, of the exclusive and private Antelope Club, one of the most prestigious clubs in their city. The club includes some lawyers and judges but it is not a club only for lawyers; its membership also includes business leaders, journalists, educators and others. The club has no openly gay members, and it recently voted that openly gay individuals are not eligible for membership. The club provides a social setting for meals, relaxation, golf and tennis. Neither Winthrop nor Leonard engage in any judicial or legal activities at the club. Who, if anyone, must resign from membership? A. Only Winthrop B. Only Leonard C. Both Winthrop and Leonard D. Neither Winthrop nor Leonard

A. Only Winthrop

Sasha is an associate in the law firm of Garrett & Glass, LLP. Recently, Sasha discovered that Gerry Glass, one of the partners, had stolen large sums of money from Healthwest, a client organization. In one case, for example, Gerry was defending Healthwest in a personal injury suit. Gerry reached a settlement under which Healthwest would pay the plaintiff $100.000. but he told the CEO of Healthwest that the organization was obliged to pay $200,000. Healthwest sent Gerry a check to cover the settlement. Gerry deposited the check in a firm account, paid the plaintiff $100,000, and wrote himself a check for the remainder. Gerry did this secretly. His law firm billed the client for legal fees as usual. Gerry repeated this same pattern in a series of cases over a seven-year period. The evidence that Gerry has stolen funds from Healthwest is indisputable. Sasha explained the whole situation to the law firm's general counsel. He instructed Sasha to keep the matter to herself. The general counsel, who is an old friend of Gerry's, said that he didn't want to blow the matter out of proportion and hurt Gerry unnecessarily.? He said he would talk with Gerry about it and make sure that the clients were reimbursed and that the situation would not recur. Which of the following correctly states Sasha's professional duty in this situation? A. She must report the misconduct to the disciplinary authorities if she can do so without disclosing any client confidences. B. She must report the misconduct to the disciplinary authorities even if doing so would reveal client confidences. C. Sasha has reported the matter to a senior supervisory lawyer in the firm and has no further obligations. D. Sasha has no further ethical duties in this case because she has received assurances that the matter will be resolved satisfactorily.

A. She must report the misconduct to the disciplinary authorities if she can do so without disclosing any client confidences.

In 2009, Anita, a criminal defense lawyer, represented Liam, an immigrant from Australia who was charged with selling cocaine. Under state law, if convicted, Liam could have been sentenced to five years in prison. The district attorney offered a deal providing that if Liam pled guilty, he would be sentenced to serve a term of 14 months. A criminal defense lawyer must give a client accurate advice about the potential adverse legal consequences of entering a guilty plea. Even so, without researching the potential immigration consequences of this guilty plea, Anita recommended that Liam accept the offer, and Liam did so. At the end of the 14 months, the state turned Liam over to federal immigration authorities, who deported him to Australia based on the criminal conviction. Liam cannot show that he would not have been convicted of a deportable offense if he had gone to trial. In 2010, the Supreme Court decided the Padilla case, holding that a conviction based on prejudicially incompetent plea-bargaining advice from a lawyer could be reversed. The highest court of the state has held that the Padilla decision does not apply to advice given before that decision. May Anita be subject to discipline? A. Yes, because Anita failed to investigate the potential immigration consequences of pleading guilty and failed to advise her client about that. B. No, because Liam cannot show that if he had gone to trial, he would not have been convicted of a deportable offense. C. No, because the state's highest court has held Padilla not to be retroactive. D. No, because Liam is not a U.S. citizen, and an American lawyer should not be disciplined for conduct in representing him.

A. Yes, because Anita failed to investigate the potential immigration consequences of pleading guilty and failed to advise her client about that.

Carla, a graduating law student who is in the United States on a student visa, is having lunch with her friend Alice, an environmental lawyer who graduated from the same law school the previous year and who was just admitted to the bar. Carla knows that during law school, Alice did a summer internship at a small immigration law firm, where she helped companies that were seeking employment visas for some of their workers. Carla tells Alice that her student visa is about to expire, but that she has just been offered a job at a law firm. She asks whether the law firm could sponsor her for a visa. Alice responds, "Unfortunately, I don't believe that law firms can sponsor people who are straight out of law school for employment visas. My advice would be to go back to your home country for a year and apply for jobs from there.? Carla, who reasonably believes that Alice was providing correct legal advice, turns down the law firm job and returns to her home country. A few months later, she learns that Alice's advice was incorrect. The law firm could have sponsored her for a work visa. She tries to get the job back, but the firm informs her that it hired someone else and can no longer hire her. Alice's advice was negligent and was the cause of Carlas return to her home country and failure to get the job. Alice never signed a retainer, paid a fee, or otherwise agreed to be Carlas lawyer. Carla sues Alice for negligence. May a court grant judgment against Alice for malpractice? A. Yes, because Carla reasonably believed that Alice was providing correct legal advice B. Yes, because a person whose only prior experience with a field of law was a summer job should not give legal advice in that field. C. No, because no consideration was given in exchange for the advice. D. No, because Carla knew that Alice was an environmental lawyer, not an immigration attorney.

A. Yes, because Carla reasonably believed that Alice was providing correct legal advice

Carson is admitted to practice only in state A. State A has adopted Rule 8.4 of the Model Rules of Professional Conduct. Carson commits a minor fraud in state B against a citizen of state B, and is prosecuted for this misdemeanor offense and punished in state B. Is Carson subject to discipline in state A? A. Yes, because Carson's conduct would violate state A's ethics code, regardless of where the conduct occurs. B. Yes, because Carson was convicted of the misdemeanor offense in a court of law. C. No, because Carson's conduct did not occur in state A and has no impact on the citizens of state A. D. No, because the matter has been criminally prosecuted in state B, so a disciplinary proceeding would impose double jeopardy on Carson.

A. Yes, because Carson's conduct would violate state A's ethics code, regardless of where the conduct occurs.

Milan graduated from law school, took the bar exam in state A, passed the exam, and was admitted to the bar in state A. He maintains active membership even though he is neither practicing law, nor living in state A. Milan bought a small computer consulting firm in neighboring state B and ran it successfully for a few years, earning a good living. Then Milan decided to sell the business. He found a buyer who purchased the business. In the course of their discussions about the business, Milan represented that the business had been twice as profitable as it actually was. After a few months of operating the business, the buyer sued Milan for damages for fraud and to rescind the contract. That lawsuit is pending. Is Milan subject to discipline in state A? A. Yes, because Milan lied to the buyer about the value of the business, even though his dishonesty took place in state B. B. Yes, because the terms of the agreement (including the value of the business) were not fair and fully disclosed to the buyer, as required by Rule 1.8 (a), nor did the buyer give informed consent. C. No, because Milan was not practicing law and this deal was not related to law practice. D. No, because Milan's acts have not yet been found to be fraudulent by a court.

A. Yes, because Milan lied to the buyer about the value of the business, even though his dishonesty took place in state B.

Attorney Cherie represents Patience in a suit against her landlord for failure to do adequate remediation after a flood caused mold growth in the attic. Patience is an anthropologist who is writing a novel about a forensic anthropologist who solves murder mysteries. Patience is low on cash, but her upcoming book is expected to be a bestseller. Cherie proposes a fee arrangement in which Patience will pay Cherie a percentage of the royalties from her book as the fee for the landlord-tenant dispute. She discusses the advantages and disadvantages of this arrangement. Patience agrees to pay the legal fee in this manner. Cherie then sends Patience a letter that includes a clear explanation of the terms of the arrangement, which are fair and reasonable. The letter also advises Patience that it is desirable for her to seek independent legal advice before signing this agreement. A few weeks later, Patience signs the letter to indicate her consent. May Cherie and Patience agree that the legal fee will consist of a percentage of Patience's royalties from the book? A. Yes, because Cherie complied with the disclosure and other requirements that govern business transactions between lawyers and clients. B. Yes, because Patience's payment of the royalties to Cherie is not contingent upon her winning the case. C. No, because a lawyer may not enter into a fee agreement with a client that gives the lawyer media or literary rights. D. No, because the litigation is ongoing. Once the litigation has ended, Cherie and Patience may make an agreement to give Patience a share of the royalties.

A. Yes, because Cherie complied with the disclosure and other requirements that govern business transactions between lawyers and clients.

Bobby, a former employee of Talmart, Inc., has retained Ashley to bring an employment discrimination action against Talmart, claiming that his firing last year was unlawful discrimination on the basis of religion. Ashley is investigating the matter but has not yet initiated a suit. Ashley wants to interview Chi, a former Talmart employee who left the company two years ago. Chi was Talmart's Vice President for Human Resources; she was responsible for hiring, promotion, and discharge of employees. Bobby told Ashley that he was fired after he complained about Talmart's policy prohibiting employees from wearing turbans while at work. Bobby says his Sikh faith requires him to wear a turban. Ashley wants to contact Chi, and ask questions about Talmart's turban policy, without first informing Ned, the lawyer who Ashley knows represents Talmart in all employment-related litigation. Assuming that Ashley would identify herself to Chi as Bobby's lawyer, may she interview Chi without seeking permission from Talmart's lawyer? A. Yes, because Chi no longer works for Talmart. B. Yes, because Ashley has not yet initiated litigation, so Talmart is not a party to a suit by Bobby. C. No, because Ashley knows that Ned represents Talmart in all of its legal employment matters. D. No, because Chi was responsible for hiring and promotion at Talmart and therefore is a person whose admissions may be imputed to Talmart for purposes of civil liability.

A. Yes, because Chi no longer works for Talmart.

Attorney Pavel is a criminal defense lawyer representing Denny, who is charged with larceny. Denny, a pro bono client, is alleged to have stolen a piece of jewelry from a jewelry box during a house party. In an unrelated matter, Pavel is defending Betty, for a fee, against a criminal charge of arson. Both cases are assigned to be heard by a single judge. During Denny's trial, the prosecutor calls Betty to the stand. She testifies that she was at the party in question and saw Denny go in and out of the room where the alleged theft occurred. She further testifies that she never went into that room. Earlier, Denny had told Pavel that he saw Betty go into the room, and he suggested that she may be the thief. Pavel intends to cross-examine Betty on this point. Is there a conflict of interest? A. Yes, because Denny's interests are directly adverse to Betty's. B. Yes, because Betty is a paying client. C. No, because the representation does not involve the assertion of a claim by Denny against Betty or vice-versa. D. No, because Pavel's cross-examination of Betty is impliedly authorized to carry out the representation.

A. Yes, because Denny's interests are directly adverse to Betty's.

Ellen wants to hire Donna, an attorney, to handle her divorce case. Ellen works as a saleswoman in a department store, earning $26,000 a year. Donna proposes to charge a fee of $300 per hour, which is not an unusual hourly rate for divorce work in Ellen's community, although some lawyers charge less. Donna discloses her fee and all expenses for which Ellen will be responsible in writing, and Ellen signs the writing. May Donna charge this fee? A. Yes, because Donna informed Ellen, before starting to work, that her fee will be $300 per hour and listed the expenses for which Donna will be responsible. B. Yes, because the notice of the fee arrangement was in writing. C. No, because Donna did not give Ellen a good faith estimate of the likely total fee. D. No, because some other lawyers in the community charge less than $300 per hour for divorce work.

A. Yes, because Donna informed Ellen, before starting to work, that her fee will be $300 per hour and listed the expenses for which Donna will be responsible.

Mayra, an attorney licensed in Indiana with a law office in Gary, Indiana, had a business office in Illinois. In her Illinois business, she imported maple syrup from Canada and sold it to distributors in California. Her importing work was unconnected with the legal work that she did in Indiana. Two years ago, she deliberately misrepresented the grade of a shipment of syrup and thereby overcharged her U.S. customers. When they discovered the fraud, they confronted Mayra, who settled with them out of court for an undisclosed sum. The matter was reported in the trade press. One of her customers sent a copy of the article to the Indiana bar disciplinary authority. Is Mayra subject to discipline in Indiana based on these events? A. Yes, because Mayras conduct involved dishonesty. B. Yes, because the matter was not merely reported in the press; a customer made a complaint to the Indiana bar. C. No, because Mayra did not do anything wrong in Indiana. D. No, because Mayras conduct had nothing to do with her work as a lawyer, and she has settled with her customers.

A. Yes, because Mayras conduct involved dishonesty.

Yvonne was seriously injured in a parking lot accident at the Woodbridge Supermarket, when a Woodbridge delivery truck, driven by a Woodbridge employee, rammed her car. The Woodbridge driver claimed, at the time, that Yvonne drove right in front of him and that the accident was her fault. Yvonne hired Kate, a lawyer, to sue the Supermarket. Kate has learned that Nora, a cashier employed by Woodbridge, witnessed the accident because she was standing outside the store during a ten-minute work break. Nora has no responsibility for driving delivery trucks or maintaining the parking lot. Kate wants to interview Nora, but she suspects that Woodbridge's lawyers (if asked) would decline permission for her to conduct this interview and would insist that she initiate formal discovery. Kate does not want to incur the expense of discovery without having an idea of what Nora would say. Kate is not planning to give Woodbridge's lawyers any prior notice that she is interviewing Nora, much less ask their permission, and she is not planning to advise Nora that she has a right to consult either Woodbridge's lawyers or a lawyer of her own choosing before talking to Kate. May Kate interview Nora, away from Woodbridge's premises, under these conditions? A. Yes, because Nora is not a managerial employee of Woodbridge and has no responsibility for driving delivery trucks or maintaining the parking lot. B. No, because she must first give Woodbridge's lawyers enough advance notice that they could seek a court order preventing the interview. C. No, because before interviewing Nora she must advise Nora that she has a right to consult Woodbridge's lawyers. D. No, because before interviewing Nora, she must advise Nora that she has a right to consult with a lawyer of her own choosing.

A. Yes, because Nora is not a managerial employee of Woodbridge and has no responsibility for driving delivery trucks or maintaining the parking lot.

Seymour owns and manages a small dairy farm. Recently, Seymour discovered that his fertilizer supplier had been mislabeling its fertilizers as organic when, in fact, they were not. Seymour has had to recall many of his products that were labeled as organic, at great expense, and his reputation has suffered as a result. He wants to sue the supplier for damages. He meets with attorney Gemma to see whether she will represent him. Gemmas usual fee is $200 per hour. Knowing that Gemma is one of the best lawyers in town, Seymour offers to pay her $250 per hour for her services. Gemma has the time and requisite knowledge and experience in the field to represent Seymour. However, as a vegan, she would find it morally repugnant to represent a dairy farmer. May Gemma refuse to represent Seymour? A. Yes, because a lawyer is not obliged to accept a client whose cause the lawyer regards as repugnant. B. Yes, because Seymour is not indigent. C. No, because lawyers may not refuse to accept clients simply because they disagree with them. D. No, because representing Seymour will not impose an unreasonable financial burden on Gemma.

A. Yes, because a lawyer is not obliged to accept a client whose cause the lawyer regards as repugnant.

Parnik, a licensed attorney and state prosecutor, was arrested for stalking his ex-girlfriend. Parnik did follow his girlfriend on numerous occasions, spied on her through the windows of her house, and called her repeatedly after she had asked him not to do so. Nevertheless, a jury acquitted Parnik of the charges. Is Parnik subject to discipline for stalking his ex-girlfriend if the bar disciplinary authorities determine that he committed a criminal act that reflects adversely on his fitness to practice law? A. Yes, because a lawyer who commits a criminal act can be disciplined for it whether or not the lawyer was convicted of a crime. B. Yes, because Parnik is a prosecutor, so he has a special responsibility beyond that of other lawyers to show respect for the law and the legal system. C. No, because the stalking occurred in Parniks private life and was not related to the practice of law. D. No, because he was acquitted of the charges, so no criminal act has been proven

A. Yes, because a lawyer who commits a criminal act can be disciplined for it whether or not the lawyer was convicted of a crime.

Kwame has represented Samantha for many years. Samantha has a daughter named Rosalind and a granddaughter named Celine, four years old. Kwame and Samantha meet to discuss Samanthas will. Samantha expresses doubts about leaving all her money to Rosalind. She reports that Rosalind's new husband, Matthew, is physically abusing Celine -she has seen him hit Celine. She shows Kwame some recent photos that she took of Celine. The pictures reveal bruises. The evidence of abuse is unambiguous. A state child abuse reporting statue requires any person who suspects child abuse to report it to the states department of social services. The law, which does not exempt lawyers from reporting, authorizes criminal penalties for failure to report, Kwame counsels Samantha to report her suspicions to department of social services, but Samantha does not want to do it and does not want Kwame to do so either. Must Kwame report what Samantha has told him? A. Yes, because he is required to do so by state law and Rule 1 6 does not excuse lawyers from having to obey state law. B. Yes, because if he does not report the abuse, Celine might be seriously injured or killed, and Rule 1.6 therefore requires the revelation. C. No, because Rule 1 6 requires lawyers to maintain the confidences of their clients. D. No, because Rule 1.6(b) permits but does not require lawyers to reveal confidences in certain circumstances, such as this one.

A. Yes, because he is required to do so by state law and Rule 1 6 does not excuse lawyers from having to obey state law.

Mindy represents RenT V, which leases television sets by the month to people who do not have enough money to buy televisions. Some of its customers, represented by Ross, brought a class action against it for failure to comply with the federal Truth-in-Lending Act. That law provides that the lawyer for a prevailing plaintiff may recover attorney's fees from the defendant in addition to the judgment. RenTV is willing to pay $120,000 to the members of the class to settle the suit. But it does not want any of the settlement money to be paid to Ross, the plaintiff's attorney, because Mindy and RenT V believe that he will just use the money to bring more troublesome litigation against RenTV or other companies owned by RenT V's parent company. May Mindy condition the settlement offer on Ross's waiver of attorney fees? A. Yes, because if the plaintiff class accepts the offer and Ross receives no fee, that's Ross's problem, not Mindy's. B. No, because Mindy is receiving a fee from her client, RenT V. C. No, because the offer would create a conflict of interest between Ross and his clients. D. No, because the federal policy of encouraging enforcement of the Truth-in-Lending Act overrides the policy of allowing parties to settle on any terms on which they agree.

A. Yes, because if the plaintiff class accepts the offer and Ross receives no fee, that's Ross's problem, not Mindy's

Donald was suspected of having robbed a liquor store. Shortly after he was arrested, he demanded to see his lawyer, Craig. The police did not attempt to question Donald, and Craig arranged for him to be released on bail at his arraignment, after Donald was read the charges filed against him and given a court date. Craig filed papers informing the court and the prosecutor's office that he would be representing Donald in the matter. Brenda, a prosecutor, was assigned responsibility for Donald's case. A state statute authorizes prosecutors to investigate criminal cases by questioning witnesses or authorizing police officials to do so up to the point at which charges are filed. Brenda visited Donald's home a week after he was released, identified herself as the prosecutor in the case, and asked if she might ask him some questions. Donald said he would answer her questions because he was innocent of the charges and therefore had nothing to hide. Brenda did not make any false or misleading statements to Donald. On the basis of information from his answers, Brenda was able to contact and interview other witnesses. She did not offer into evidence any information that she received from Donald. Is Brenda subject to discipline? A. Yes, because she did not first obtain Craig's consent to talk to Donald. B. Yes, because she did not first notify Donald of his right to have counsel present at the interview. C. No, because she did not make any misleading statements to Donald. D. No, because prosecutors are authorized by law to interview defendants.

A. Yes, because she did not first obtain Craig's consent to talk to Donald.

Armand DuBois was elected as a state court judge seven years ago. This year, he is up for re-election. His state has adopted the Model Code of Judicial Conduct, which bars a candidate for judicial office from personally soliciting or accepting campaign contributions. A rule of the state supreme court provides that violation of the Code may be punished by that courts judicial conduct committee, which may impose sanctions ranging from a reprimand to suspension of judicial duties. Judge DuBois personally accepted a $500 contribution from his close friend and tennis partner, Stanley Baskin. Is he subject to discipline by the judicial conduct committee? A. Yes, because the Supreme Court's Citizens United decision does not apply to judicial campaigns. B. No, because under the Citizens United case, Judge DuBois has a First Amendment right to accept campaign contributions. C. No, because a judge may be punished only by impeachment and conviction by the legislature. D. No, because contributions from close friends and relatives to judicial candidates are permitted.

A. Yes, because the Supreme Court's Citizens United decision does not apply to judicial campaigns.

Sol practices tax law, but his expertise is in giving tax advice rather than handling litigation. Recently, Sol provided personal tax advice to businessman Bill. Bill then asked Sol to look at another tax matter involving a deduction denied by the Internal Revenue Service. Bill wants Sol to litigate the matter in the tax court. Sol mentioned to his friend Al, a litigator in a different firm, that one of his clients wants him to take on a litigation matter. Al responded, "Great! Just tell your client that you will do it with co-counsel. I will charge Bill 33 percent of the recovery, and I'll give you half of what he pays us. We'll both enter appearances and sign the papers, but you can leave everything to me. It will be as if we were in a law partnership together!" Sol believes that Al is an excellent litigator capable of doing a good job for Bill. sol and Al agree to accept any liability for mistakes. The overall fee they intend to charge is reasonable. Bill gives his informed consent to Al's association with Sol and to the proposed fee and to how Sol and Al will split it, and he signs a writing to that effect. May Sol and Al enter into this split-fee arrangement? A. Yes, because the arrangement complies with all of the relevant rules. B. Yes, because there are no restrictions on lawyers sharing fees. C. No, because Sol is not competent to litigate the matter. D. No, because the division is not in proportion to the actual work performed by each lawyer.

A. Yes, because the arrangement complies with all of the relevant rules.

Jabari is an associate at a law firm in state A. He is admitted to practice only in state A. Jabari goes to state B to litigate a matter on behalf of a client. He is admitted pro hac vice in stale B. Jabari violates an ethical file That has been adopted in state B requiring disclosure of client perjury to the judge. State A has not adopted any similar rule. Could Jabari be disciplined in state A even though state A has not adopted the rule that Jabari violated? A. Yes, because the disciplinary authorities in state A would apply the rules adopted in state B in this case. B. Yes, because state A is the only state in which Jabari is a member of the bar, so state A is solely responsible for overseeing his conduct. C. No, because Jabari's conduct did not take place in state A. D. No, because Jabari's conduct does not violate the ethics code in state A.

A. Yes, because the disciplinary authorities in state A would apply the rules adopted in state B in this case.

Carlos, a sole practitioner, represents Sharky Products, Inc, a property owner that disputes In adjoining owner's right of access to an alleyway between their buildings. Carlos sent a letter to the other property owner, Berry's Beauty Supply Co, to explain the problem. He received a reply from Berry's lawyer, Eleanor, requesting that Carlos withdraw from representation of Sharky, Eleanor claimed that Carlos was prohibited from handling this matter because five years ago, Carlos represented Matthew, the president and sole owner of Berry's in a claim against the manufacturer of a lawn care product that he used at his home. The matter took three hours of Carlos time and Matthew was satisfied with the resulting settlement. Carlos has had no contact with Matthew since then. May Carlos continue to represent Sharky Products, Inc.? A. Yes, because there is no substantial relationship between the two matters. B. Yes, because he only worked on Matthew's case for three hours. C. No, because his representation of Sharky is materially adverse to his former client. D. No, because Matthew is the president and sole owner of Berrys.

A. Yes, because there is no substantial relationship between the two matters.

Danielle is a lawyer. Ted, the son of her brother Matthew, is very ill with a life-threatening condition. Ted must get a certain medication that costs $1,400 in the next 24 hours or else he will go into kidney failure. Matthew has no cash, but he will receive a large sum from the sale of his home in one week. Matthew has no other possible source of funds until then unless Danielle gives him a short-term loan. Danielle is flat broke (in fact, she is deeply in debt), but there is $120,000 in Danielle's trust account. This amount is the proceeds of a personal injury settlement that Danielle will distribute to her client Van. She represented him on a pro bono basis, so she is not owed a fee from the settlement. Van is in prison for the next few years. Danielle has agreed to hold the funds for Van in the interim and to make mortgage payments on his house as they become due. Danielle lends Matthew $1,400 from this account for Ted's medication. She is unable to ask for Van's approval because he is in solitary confinement because he talked back to a prison guard and is not allowed to receive phone calls. Danielle, confident that Van would approve, sends him a letter explaining her intention to make the loan to her brother. A week later, Matthew receives the proceeds from the sale of his house. He repays Danielle, who replenishes the client trust account. Van later writes Danielle that he is glad that she was able to make the loan to her brother. There is no resulting delay in the distribution of the settlement funds to Van or in the making of mort- gage payments. Is Danielle subject to discipline? A. Yes, even though she promptly informed Van that she borrowed the money and he ratified her prior action. B. No, because she promptly informed Van that she loaned out a chunk of his money and he ratified her prior action. C. No, because there was no delay either in making the payments on Vans mortgage or in the distribution of the settlement funds to Van. D. No, because she did this to prevent reasonably certain death or substantial bodily harm to a child.

A. Yes, even though she promptly informed Van that she borrowed the money and he ratified her prior action.

Geraldine, an attorney in a legal aid program, provides pro bono representation to Amber, who is indigent, disabled, and homeless, in litigation against Mike's Job Counseling Service. Amber had paid $300 to Mikes, which did not give her any job leads or help. Mikes has recently been exposed in the local paper for not actually having helped anyone to get a job. Winter is approaching, and it has become increasingly difficult for Amber to live on the streets. She is in danger of freezing to death. All of the homeless shelters in the area are full. Geraldine cares about Amber and wants to keep her from freezing. Which of the following statements is correct? A. Geraldine may loan Amber $500 so that she can rent a modest room. B. Geraldine may give Amber $500 so that she can rent a modest room. C. Both A and B are correct. D. Neither A nor B is correct.

B. Geraldine may give Amber $500 so that she can rent a modest room.

Which one of the following statements is correct? A. American lawyers must comply with ABA Model Rules of Professional Conduct or face discipline. B. No judicial conduct code binds the justices of the United States Supreme Court. C. The Restatement of the Law Governing Lawyers provides a synthesis only of the state ethics codes and the case law interpreting those rules. D. A state ethics code may not impose requirements on lawyers employed by the federal government that conflict with duties imposed by federal agency regulations.

B. No judicial conduct code binds the justices of the United States Supreme Court.

Spencer, a businessman, calls Kai, an attorney. He says he is looking for a lawyer so that he can sue his former business partner, Max, with whom he has had a falling out. He asserts that he has proof that Max was stealing money from his company and covering it up by keeping two sets of books. Spencer says he has photos of several pages from both sets of books, which he obtained by sneaking into Max's house, where he found the books in the study and photographed them with his cell phone. Kai makes an appointment to meet with Spencer the following week to discuss possible representation. The day after the phone call, Max calls Kai to ask for representation because he thinks Spencer is going to sue him. Max offers Kai a generous flat fee for the work. Kai would like to represent Max. May Kai represent Max without Spencer's consent? A. No, because an attorney may not accept representation of a person where the attorney's representation has first been requested by a potential adversary of that person. B. No, because Spencer has given Kai information that could be used adversely to Spencer if Kai represents Max. C. Yes, because there is no conflict of interest, since Kai has not yet agreed to represent Spencer. D. Yes, because Spencer's actions were not lawful.

B. No, because Spencer has given Kai information that could be used adversely to Spencer if Kai represents Max.

Warren, a lawyer, represented Carrie, a state university student, in a lawsuit challenging her state's restrictive voter identification law. The retainer agreement specifies that Warren would not charge Carrie a fee but may keep, as his fee, any counsel fees that the court requires the state to pay. A state law allows out-of-state students to vote, but it requires voters to supply photo identification. Pursuant to regulation, the state accepts many forms of picture identification, including passports, in-state driver's licenses, and gun licenses, but it does not accept out-of-state driver's licenses. Carrie only has an out-of-state driver's license. After Warren filed a civil rights suit in federal court on her behalf, the state changed its regulation to allow out-of-state driver's licenses to qualify as identification. The state attorney general explained to the press, "We didn't think that the court would sustain our regulation." Warren has signed an agreement with the state attorney general in which he agreed to withdraw the lawsuit and the attorney general agreed, on behalf of the state, to accept out-of-state driver's licenses as photo identification for purposes of voting for at least 25 years. Now Warren plans to request the court to award him counsel fees based on the time he spent researching the law and filing the suit. Should he expect to obtain court-awarded fees? A. Yes, because the Civil Rights Attorney's Fees Act of 1976 (the Fees Act) permits federal judges to shift fees to prevailing parties in civil rights cases against state actors, and judges usually do so. B. No, because the settlement was not embodied in a court judgment. C. No, because the Supreme Court has held the Fees Act to be unconstitutional. D. No, because under the American Rule, each party pays his own attorney's fee.

B. No, because the settlement was not embodied in a court judgment.

After she graduated from law school, Dania worked for the Securities and Exchange Commission (SEC) for three years. During that time, she worked on securing an indictment in a large securities case involving seven defendants who collaborated on an insider trading scheme. At the end of three years, she moved on to a position at one office of a large private law firm whose practice includes defense of securities fraud cases. A month after she started work at the firm, Dania learned that a partner in the firm was representing one of the defendants in the securities case that Dania had worked on while she was at the SEC. This did not emerge in the preliminary conflicts screening before she was hired because Danias work focused on three of the other defendants in the suit. The basic facts were the same, but the targets were different. What should the firm do to enable it to represent the defendant in the securities fraud case? A. Instruct Dania not to reveal to anyone in the law firm anything she learned about the case while she was at the SEC. B. Screen Dania from any participation in the matter, prevent her from receiving any extra pay related to the matter, and give written notice of the potential conflict to the SEC C. Transfer Dania, while the litigation is pending, to a different office of the firm, in another city. D. Discharge Dania, because that is the only way in which the firm could both comply with the ethics rules and continue to represent its client.

B. Screen Dania from any participation in the matter, prevent her from receiving any extra pay related to the matter, and give written notice of the potential conflict to the SEC

Attorney Alton recently filed a medical malpractice action on behalf of Juan against Metropolitan Hospital, where Juan was sexually assaulted by a staff member while he was being treated for a psychotic episode in the psychiatric ward. Juan was traumatized by his experience at the hospital. In a telephone call before their first meeting, Alton learned that Juan suffers from post-traumatic stress disorder and needs a social worker to be present in all stressful situations. Alton therefore asked Juan to bring his social worker, Pierre, to all of their metings. Juan pays Pierre to assist him, and he brought Pierre to all of his meetings will Alton. The lawyers for Metropolitan Hospital have learned about Pierres presence during the meetings between Juan and Alton. The hospital is seeking discovery of the conversations that took place during those meetings. Which one of the following statements is correct? A. The attorney-client privilege is likely to protect the conversations from discovery because Alton, rather than Juan, suggested that Pierre should participate in the meetings. B. The attorney-client privilege would have protected the conversations between Alton and Juan from discovery despite Pierre's presence even if Juan had brought him to the meetings without any suggestion from Alton. C. The conversations are unlikely to be protected from discovery by the attorney-client privilege because Pierre was hired by Juan rather than Alton. D. The attorney-client privilege is unlikely to protect from discovery any conversation between Alton and Juan because Pierre was present at their meetings.

B. The attorney-client privilege would have protected the conversations between Alton and Juan from discovery despite Pierre's presence even if Juan had brought him to the meetings without any suggestion from Alton.

Grace is an associate in a law firm. Her firm bills clients by the hour. When she began working for the firm, she signed a statement in which she agreed to abide by firm policy. The firm policy stated in part: "All misconduct shall be reported promptly to the managing partner and shall not be revealed to anyone else in the firm or outside the firm.? Grace discovers that Arthur, the partner who supervises her work, always alters her time sheets by multiplying the number of hours she reported working by 1.2, thereby claiming to the clients that she spent 20 percent more time on each project than she actually spent, and billing the clients accordingly. Grace reports the misconduct to the managing partner, who tells her that she need not concern herself with the conduct of her superiors. He takes no action. Would Grace be subject to discipline if she failed to report Arthur's conduct to the disciplinary authorities? A. Yes, because otherwise she might be blamed for the over-billing. B. Yes, because Arthur's conduct involves a violation of the rules that raises a substantial question as to his honesty. C. No, because she is contractually bound to report misconduct only to the managing partner in accordance with the firm's policy, and the managing partner has a duty to report the conduct to the bar authorities. D. No, because it is unlikely that she would be disciplined for failure to report Arthur's conduct, and if she does report the matter outside of the firm, the firm will fire her.

B. Yes, because Arthur's conduct involves a violation of the rules that raises a substantial question as to his honesty.

Five years ago, attorney Barry worked at a large law firm with offices across the country. One of the firm's partners defended Panko, Inc., an appliance manufacturer, against allegations that its toasters had a defect that would cause many of them to catch fire. Barry was not involved in that litigation and did not learn anything about it during his time at the firm. Barry has since opened up a private practice of his own. Ted comes to Barrys new office seeking legal help. Ted tells Barry that his Panko toaster recently overheated and caused a house fire. He would like to sue Panko for damages. May Barry represent him a suit against Panko without Panko's informed consent? A. Yes, because there was not a substantial risk that confidential factual information as would normally have been obtained in the prior representation of Panko would materially advance Ted's position in the subsequent matter. B. Yes, because Barry did not actually acquire confidential information that is material to Ted's lawsuit while he was working at his old law firm. C. No, because the matters are the same or substantially related. D. No, because Ted's interests are materially adverse to those of Panko.

B. Yes, because Barry did not actually acquire confidential information that is material to Ted's lawsuit while he was working at his old law firm.

Attorney Eli has represented Jacques, an elderly but lucid man, in his legal matters for the past eight ears. He continues to do so from time to time. The legal matters with which Eli has assisted Jacques have been minor; the most Jacques ever paid Eli was $1,500. One day, Jacques tells Eli that as a token of his appreciation for his years of service and friendship, he would like to give Eli his mint condition 1970 Chevrolet Chevelle, a car Eli knows to be worth over $70,000. Eli refers Jacques to another lawyer who can prepare the documents transferring the title to the vehicle to Eli. May Eli accept the gift A. Yes, because there are no restrictions with respect to receiving unsolicited gifts from clients. B. Yes, because Eli referred Jacques to another lawyer who will prepare the documentation to effectuate the transfer of the title to the vehicle. C. No, because Eli did not first arrange for a guardian ad litem to be appointed for Jacques. D. No, because a lawyer may not accept a substantial gift from a current client, and the car would be considered a "substantial" gift.

B. Yes, because Eli referred Jacques to another lawyer who will prepare the documentation to effectuate the transfer of the title to the vehicle.

Ella, a public defender, represented a college student named Kenneth. Kenneth was driving his brother Thomas car and was stopped for going through a red light. The officer spotted what looked like cocaine and drug paraphernalia on the front seat and arrested Kenneth. When the substance was tested, it turned out to be cocaine. Against Ella's advice, Kenneth told Ella that he planned to plead guilty. Ella writes a blog in which she regularly recounts her experiences as a public defender. After Kenneth was released pursuant to a negotiated deal for deferred prosecution, she wrote on her blog: * "#126409 (the client's jail identification number): This stupid kid is taking the rap for his drug-dealing dirtbag of an older brother because 'hes no snitch: I managed to talk the prosecutor into treatment and deferred prosecution, since we both know the older brother from prior dealings involving drugs and guns. My client is in college. Just goes to show you that higher education does not imply that you have any sense. Jail identification numbers are not available to the general public. Is Ella subject to discipline? A. Yes, because Ellas comment about the case did not reveal any confidential information, but it disparaged her unnamed client. B. Yes, because Ella revealed confidential information. C. No, because Ella did not name her client. D. No, because the only identifying information in Ellas disclosure was the non-public jail identification number.

B. Yes, because Ella revealed confidential information.

Laila was a pedestrian who was hit by a car. Her best friend Joan, who is a nurse, visited her in the hospital. Neither Joan nor Laila knew the driver of the car. Joan later discussed the accident with her friend Craig, who is an attorney. Craig would like to represent Laila, on a contingent fee basis, in a claim against the driver of the car. Craig tells Joan that if she recommends him to Laila, and Laila retains him, Craig will pay Joan 10 percent of his share of any recovery that Laila collects. He does not tell Laila about this part of the arrangement. Is Craig subject to discipline? A. Yes, because he did not disclose the arrangement to Laila. B. Yes, because Joan is not a lawyer. C. No, because Joan has no connection with the driver of the car and there is therefore no conflict of interest. D. No, because he offered Joan 10 percent of his own fee, rather than 10 percent of the recovery.

B. Yes, because Joan is not a lawyer.

Josh, a third year law student, is planning to apply for admission to the bar. He just looked at the questionnaire of the National Conference of Bar Examiners, which he will have to submit. It asks: Have you ever been dropped, suspended, warned, placed on scholastic or disciplinary probation expelled, requested to resign, or allowed to resign in lieu of discipline from any college or University (including law school), or otherwise subjected to discipline by any institution or requested or advised by any institution to discontinue your studies there? Four years ago, a residential advisor found alcoholic beverages in Josh's dormitory room in violation of college rules. At a meeting with a dean, Josh agreed to be put on probation for one semester, during which he would be allowed to take classes as usual. If he made it through the semester without any more infractions, the incident would be expunged from his record. He completed the semester without incident, and the matter was therefore expunged from his college record and not recorded on his transcript. His law school asked a similar question on its admission application, and he answered "no" to the question. Should Josh reveal the above-described incident in his character and fitness questionnaire? Which of the answers below is most accurate? A. Yes, because he did not disclose the incident on his law school application. B. Yes, because Josh was placed on disciplinary probation by his college, so the question calls for disclosure of the incident, and includes no exemption for "expunged" offenses. C. No, because Josh is not yet admitted to the bar, so he is not bound by the ethics code. He has the discretion to make his own best judgment about whether this should be disclosed. This minor incident has no bearing on his fitness to practice law, so there is no reason to reveal it. D. No, because it was expunged by his college, reflecting the intention of the college that the minor incident should have no future consequences.

B. Yes, because Josh was placed on disciplinary probation by his college, so the question calls for disclosure of the incident, and includes no exemption for "expunged" offenses.

Kieran, five years out of law school, accepted a job with Sato & Perlmutter, LLP. A couple of months after he started work, Miranda Perlmutter was asked to take over representation of Kasho Natural Foods in an ongoing lawsuit against the Roxbury Box Company. Kasho had fired its previous counsel after a dispute over legal fees. Miranda wants to accept Kasho as a client. However, there may be a problem. Kieran came to Sato & Perlmutter from another firm, Podkrash and Associates, LLP, where he had worked for three years. The Podkrash firm has been representing the Roxbury Box Company in the Kasho litigation for the last two years. Kieran did not work on the Roxbury matter while he worked at Podkrash. While Kieran was working at Podkrash, he dated Lance, the senior paralegal at the firm. Lance oversaw all the staff work on the Roxbury matter and talked at length about the factual and strategic issues in the case with Kieran on many occasions. For example, the two men talked about a decision of the partner in charge of the litigation that a key document that might impact Roxbury's liability was not covered by the plaintiffs' discovery request. Lance disagreed with the partner's decision, but ultimately decided to keep his mouth shut. After consultation with her firm's ethics counsel, Miranda decides to undertake representation of Kasho. However, she does not wish to terminate Kieran's employment as a way to solve any conflicts problem. Instead, she timely screens Kieran from the lawyers working on the matter, sees to it that he receives no part of the fee from the Kasho matter, and provide appropriate written notices to Roxbury about the screening procedures. Was Sato & Perlmutter permitted to undertake representation of Kasho in this manner? A. Yes, because Lance and Kieran's relationship was personal in nature. B. Yes, because Kieran was properly screened from the Kasho representation. C. No, because there is a substantial risk that confidential information that Kieran would have learned at the Podkrash firm would materially advance Kasho's position in the litigation. D. No, because this is a situation in which screening is insufficient to avoid the possibility of a conflict of interest so long as Kieran remains at the firm.

B. Yes, because Kieran was properly screened from the Kasho representation.

Attorney Roger is a solo practitioner in a small town. There are only about 70 lawyers in the town, and of those, only 20 (including Roger) handle divorce cases. One day, Roger does an intake interview with Leon, who is seeking a divorce. Leon reveals to Roger that he had several affairs during his marriage and that his wife, Reva, does not know about them. At the end of the interview, Roger tells Leon that he will let Leon know within a week whether he can accept his case. The next day, Leon calls Roger to say that he has decided to retain another lawyer. A week later, Reva comes to Roger's office seeking representation in the divorce. Recognizing her name, and before she says anything else, he asks her if Leon is her spouse. She confirms that he is. Roger immediately tells Reva that he may not be able to represent her. Before he can finish his sentence, Reva interrupts, "Is this because he came to see you already? I have already been told by five different lawyers that they cannot represent me because of a conflict of interest! I think Leon is just trying to prevent me from hiring a lawyer in this town" A few days later, Roger is able to confirm that over the course of three days, Leon did a one-hour intake interview with every divorce lawyer in town. May Roger represent Reva in the divorce? A. Yes, because Leon is only a prospective client; therefore Roger owes no duty to him. B. Yes, because Leon met with every divorce lawyer in town. C. No, because Roger received information from Leon that could be used against Leon. D. No, unless he gets Leon's informed consent, confirmed in writing.

B. Yes, because Leon met with every divorce lawyer in town.

Tax attorney Tammy represents client Lilian, who plans to buy several tickets to upcoming concerts using her student ID and sell them to non-students for a significant profit. She seeks Tammy's advice as to whether she must pay income tax on the money she expects to make. She does not know that reselling student tickets for profit is illegal in her state, and when Tammy informs her that it is, she does not go forward with the plan. Is Lilians conversation with Tammy privileged? A. Yes, because all confidential communications between lawyers and clients for the purpose of obtaining legal advice are privileged. B. Yes, because Lilian did not go through with her plan. C. No, because the crime-fraud exception to the attorney-client privilege applies. D. No, because Lilian did not know that scalping was a crime when she asked Tammy for advice.

B. Yes, because Lilian did not go through with her plan.

Use the facts from the previous question. Suppose Morgan decides not to include the malpractice language in the retainer agreement, and both she and Barbara sign the agreement. Suppose further that Morgan indeed makes a serious mistake in the course of representation. Morgan and Barbara estimate that the mistake cost Barbara $9,500. Morgan would like to settle any malpractice claim that Barbara may have against her for $12,000 without litigation. Morgan does not wish to notify her malpractice insurer of the mistake, because then her rates would go up. She advises Barbara in writing to seek independent counsel, and she tells her she can take her time finding another lawyer. Barbara declines, saying she does not need another lawyer and that she agrees with the $12,000 settlement. May Morgan go forward with the settlement? A. Yes, because Morgan's mistake cost Barbara less than $12,000. B. Yes, because Morgan advised Barbara in writing regarding the desirability of seeking independent counsel. C. No, because Barbara was not represented by another lawyer in the settlement negotiations. D. No, because the ethics code requires lawyers to report professional errors to their malpractice insurers before settling malpractice claims based on those errors.

B. Yes, because Morgan advised Barbara in writing regarding the desirability of seeking independent counsel.

Attorneys Carrie and Paige are partners in a small firm. Paige supervises the work of the firm's two associates. Over the last few years, Carrie has noticed that Paige has been intoxicated during the workday. In the last few months, Carrie has been smelling alcohol on Paige's breath during the workday at least two or three days a week. Paige continues to meet with clients and work on cases, but Carrie believes Paige's work is suffering as a result of her drinking. In the past two weeks, she has received calls from three of Paige's clients complaining that Paige either failed to show up for a meeting or failed to return a call. Last Tuesday, she found Paige passed out on the couch in her office at 11 am. There was vomit all over a client file that was on the floor near her head. Today she learned that a case that Paige was handling, to recover damages for a woman who was badly injured in an accident, was postponed for six months after Paige failed to appear for a court hearing. Carrie has tried to talk with Paige about her drinking problem, but Paige refuses to discuss it. Carrie is considering reporting Paige's behavior to the disciplinary authorities. She has not discussed this with other attorneys in the firm or with any clients. Would Carrie be subject to discipline if she fails to report Paige's conduct to the disciplinary authorities? A. Yes, because Paige has supervisory authority over associates. B. Yes, because Paige's alcohol consumption suggests that she is ill and needs medical care, but her conduct also raises a substantial question about her fitness to practice law in her current state. C. No, because Carrie should seek permission from Paige's clients before reporting misconduct, as they could be harmed if Paige is suspended from practice. D. No, because a lawyer is not required to report information relating to the conduct of the lawyer's own partners.

B. Yes, because Paige's alcohol consumption suggests that she is ill and needs medical care, but her conduct also raises a substantial question about her fitness to practice law in her current state.

Paula, a lawyer, drafted a will for Viet. The will stated that in the event of his death, his property should go to "my children." Viet and his wife Jenny had two sons. After his death, Edith, a woman with whom he had had an affair, proved through DNA evidence that he also had fathered her son Michael. She claimed that her son was entitled to one third of the estate. During probate of the will, Paula plans to testify that Viet had told her that what he meant was that his property should go to his children Gabriel and Veronique, and any children that he and Jenny later produced. Since Jenny is next of kin, Paula has obtained her informed consent to this plan. Paula would assert that she was waiving the privilege on behalf of her deceased client. Edith's lawyer objects on the ground that Paula's testimony would violate the attorney-client privilege. Should the probate judge allow Paula to testify about Viet's intent? A. Yes, because Jenny as next of kin has given informed consent. B. Yes, because Paulas testimony would fall under an exception to the privilege. C. No, because the information is privileged. D. No, because Paulas testimony would result in an injustice to Michael.

B. Yes, because Paulas testimony would fall under an exception to the privilege.

Attorney Seamus represents Rona in her purchase of an apartment in a major metropolitan city. The apartment is priced well below market value, and it seems to both Seamus and Rona that it is a very good deal. Seamus learns that the apartment is priced so low because one of the bedrooms was constructed below ground level in violation of the city's housing code. When Seamus relays this information to Rona, she gets cold feet and decides not to sign the contract. She finds another apartment and closes a purchase agreement. As it happens, Seamus is also looking to buy an apartment. The apartment that Rona declined to buy would work well for him and his family. He is also willing to buy the apartment knowing that it has a bedroom that does not meet the code requirements. He talked with Rona about his interest in the apartment. She says it would not concern her at all if he bought it. May Seamus purchase the apartment? A. Yes, because Rona consented to allow him to purchase it. B. Yes, because Rona no longer wants to buy the apartment. C. No, because he learned about the apartment in the course of his representation of Rona. D. No, because it is impermissible to use confidential information for the attorney's benefit.

B. Yes, because Rona no longer wants to buy the apartment.

The Packard Management Company manages an apartment building in which Manuel is a tenant. Tom, a lawyer, handles all legal matters for Packard. Manuel has not paid his rent for three months. Packard wants to evict him. Manuel has called Packard several times to complain about minor deficiencies in the building's services but has never asserted that he was declining to pay rent because of the inadequate services. Packard's policy directs Tom to make one effort to negotiate with a non-paying tenant for full payment within 30 days before commencing eviction proceedings. Tom's friend Ivan is Manuel's boss at Ecosystem, Inc. Ivan told Tom last week that Manuel has hired a lawyer named Serena to sue Ecosystem for race discrimination. Ecosystem has no connection to Packard. When Tom calls Manuel, he is planning to identify himself as a lawyer for Packard at the beginning of the call, but he is not planning to ask Manuel whether he is being represented by counsel with respect to his tenancy, or to advise Manuel that he has a right not to speak to Tom or to be represented by a lawyer in connection with the threatened eviction. After identifying himself, Tom is planning to ask why Manuel hasn't paid his rent and then to negotiate for prompt payment. May Tom have this conversation with Manuel? A. Yes, because no rule of professional conduct imposes restrictions on conversations between lawyers and unrepresented persons before proceedings have been initiated. B. Yes, because Tom does not know that Serena is representing Manuel in his dealings with Packard. C. No, because Tom knows that Manuel has a lawyer, Serena. D. No, because Tom is not planning to ask Manuel whether he has a lawyer with respect to his tenancy, or to advise Manuel that he has a right not to speak to Tom or to be represented by a lawyer.

B. Yes, because Tom does not know that Serena is representing Manuel in his dealings with Packard.

Leta graduated from law school one year ago and is applying for admission to the bar. While in law school, Leta accumulated $150,000 of debt from law school loans and credit card expenses related to her wedding. Upon graduation, Leta turned down a high-paying job at a prestigious law firm, choosing instead to work for $12 per hour at a public interest organization that represents indigent people in consumer and bankruptcy cases. She plans a career in consumer credit and bankruptcy law. She hopes that her work at the organization will eventually lead to a full-time public service job. Accordingly, she has not been able to begin paying off her debt. One credit card company has obtained a judgment against her for $7.000. The state bar has denied her application for admission to the bar because it finds that her conduct with respect to her finances indicates that she is not fit to practice law. Is the state bar's position legally sustainable? A. Yes, because she intends to practice consumer credit and bankruptcy law. B. Yes, because a court could conclude that her conduct with respect to her personal finances indicates that she is not fit to practice law. C. No, because the majority of law students have substantial student loan obligations, for law school. So it would be unfair to penalize her for having borrowed a large amount of money D. No. because every lawyer should aspire to provide pro bono service to clients who can't afford to pay, so a layer who does public interest work full time is not engaged in misconduct but is pursuing a career aiming to hep those who are less fortunate.

B. Yes, because a court could conclude that her conduct with respect to her personal finances indicates that she is not fit to practice law.

Eloise is a law professor in state B who maintains active licenses to practice law in states A and B. She has not practiced law in some time and does not plan to practice law in the near future. Eloise wrote and published a law review article in state B criticizing the criminal justice system in state A. The article includes long passages of plagiarized material, and includes some false statements about the law in state A. The disciplinary agency in state B is unable to decide whether the plagiarism and false statements were intentional, but it imposes a six-month suspension for this conduct. Is Eloise subject to discipline in state A? A. Yes, because many readers of the article are lawyers in state A, so the article has harmed the administration of justice in state A. B. Yes, because a lawyer may be disciplined by more than one state for a single act of misconduct. C. No, because a lawyer may be disciplined by only one state for a single act of misconduct. D. No, because the article was written and published in state B, so the disciplinary authorities in state A would not have jurisdiction.

B. Yes, because a lawyer may be disciplined by more than one state for a single act of misconduct.

Sam, a lawyer, represents Alan Pauly, a tall man who is in jail facing assault charges stemming from a fight in a bar with a short man, Don. Sam is investigating the facts. The police report mentions no witnesses other than the victim. The bartender did not see the fight but tells Sam that Toby was in the bar that Saturday night. Sam goes to Toby's house. He knocks on the door. Toby answers. After Sam confirms that the man who answered the door is Toby, the conversation goes like this: Sam: I am a lawyer for Alan Pauly, and I am investigating the fight at Harry's Bar two weeks ago this Saturday. I understand that you were there that night. Toby: That's right. It was really awful. Sam: Is a lawyer representing you in connection with this matter? Toby: No, I was not involved in the fight. Sam: Could you tell me what happened? Toby: There were these two guys. They had been drinking a lot. Then they started arguing. One was tall and the other was short. Sam: Where were they and where were you? Toby: They were at a table. I was on the barstool nearest to them. Sam: Could you hear what they were saying? Toby: I might have, but I can't remember. But I remember the fistfight well enough. I saw that pretty clearly. Sam: Have you talked to anyone else about this? Toby: No, I thought about going down to your station, but I didn't want to get involved with police. Sam wants to ask Toby who threw the first punch, the tall man or the short one. Would he be subject to discipline if that is the next thing he does? A. Yes, because he must first advise Toby that he may seek independent legal advice. B. Yes, because he must first make a reasonable effort to correct Toby's confusion about his role. C. No, because Toby is a witness, not a potential party, and does not have interests that are adverse to those of Alan. D. No, because Sam has identified himself as a lawyer for Alan.

B. Yes, because he must first make a reasonable effort to correct Toby's confusion about his role.

Attorney Tito is a sole practitioner. He represents Laura, a criminal defendant charged with stealing a diamond necklace from Matteo. Matteo and Laura went on a few dates and then the relationship went sour. Laura claims that Matteo gave her the necklace on their second date. On the day of the jury trial, in the elevator on the way to the courtroom, Tito overhears Matteo telling his friend that he gave Laura the necklace, and that the only way he could get it back was to claim that she stole it. Tito has been preparing for the trial for weeks. Tito plans to continue to represent Laura and testify in the trial as to what he heard. He has a co-counsel who could take over the trial while Tito is testifying, but Tito is much better prepared and more experienced. The transfer of primary responsibility for the trial to another lawyer would work substantial hardship on Laura. May Tito continue to represent Laura and testify in the trial? A. Yes, because Tito's testimony relates to an uncontested issue. B. Yes, because his disqualification would impose a substantial hardship on Laura. C. No, because a lawyer may not serve in the same trial as both advocate and witness. D. No, because a lawyer may not act in a trial as both an advocate and a witness unless a judge rules that such testimony is necessary to avoid a substantial hardship on a party.

B. Yes, because his disqualification would impose a substantial hardship on Laura.

Lucille, a criminal defense lawyer, receives a frantic call from Svlvia, who states that her sister, Basia, who recently immigrated from Poland, has been arrested by state police for a drug violation, and that the bail hearing will be held in less than two hours. She asks Lucille to represent Basia at the bail hearing and in her criminal case. Lucille hires Magda, a Polish-speaking interpreter, and rushes to the courthouse, where she is able to briefly meet with Basia and represent her at the bail hearing. The judge sets the bail at $2,000. Sylvia is able to pay the bail amount, and Basia is released from prison that night. The next day, Lucille meets with Basia, with Magda interpreting, and informs her that her rate for legal services is $300 per hour. She states that that rate will be used to calculate the bill for the work that Lucille did on the bail hearing, and for future work that she will do on the criminal case. Basia agrees to the rate and Lucille begins work on her case. There is no other discussion of how much Basia will have to pay. At Basia's hearing five months later, the arresting officer fails to appear, and the charges against Basia are dropped. The next month, Lucille sends Basia a bill for $1,800, including five hours of her work, plus $300 for Magdas services. Is Lucille subject to discipline? A. Yes, because she did not communicate the rate in writing or give Basia an estimate of the total fee. B. Yes, because she did not communicate the expenses for which Basia will be responsible within a reasonable time after commencing the representation. C. No, because a lawyer's rate need not be communicated in writing. D. No, because Lucille was hired on such short notice that she could not discuss compensation in advance.

B. Yes, because she did not communicate the expenses for which Basia will be responsible within a reasonable time after commencing the representation.

Attorney Maria is a divorce lawyer. Several months ago, Maria represented Jillian in her divorce from her husband, Jack. At that time, Maria charged her an hourly fee of $400 per hour. The judge granted the divorce and ordered Jack to pay her $1,000 per month child support. Recently, Jillian came to Maria seeking help, because Jack has not been paying the child support for the past 6 months. She would like Maria to help her get the $6,000 that Jack owes her. Maria wishes to charge Jillian a contingent fee of 30 percent of the recovery. She discloses the fee and expense terms and receives Jillian's informed consent in writing. May Maria charge the contingent fee? A. Yes, because Jillian is Maria's former client. B. Yes, because she obtained Jillian's consent in writing. C. No, because contingent fees are not permitted in domestic relations cases. D. No, because 30 percent is excessive because it will drain resources needed to support the couple's children.

B. Yes, because she obtained Jillian's consent in writing.

Cindy, an indigent single mother of four, sought the help of Rona, an attorney, in filing an application for emergency food stamps. Rona agreed to help Cindy for a very low, fixed fee, which Cindy paid immediately. While the application is pending, Rona realizes that even if the application is approved, Cindy's family will go hungry, because the state's welfare benefits are so low. She wants to help Cindy by giving her $100 a month, for six months, out of her own funds. May she do so? A. Yes, because Cindy is indigent. B. Yes, because the financial assistance is not being offered in connection with litigation. C. No, because Cindy did not waive any potential conflict of interest in writing. D. No, because she did not terminate the representation before providing the assistance.

B. Yes, because the financial assistance is not being offered in connection with litigation.

Jennifer is a partner at a small law firm, and she has been practicing law for eight years. During that time, she has handled primarily trusts and estates cases. Recently, however, she has taken on a domestic relations case representing a woman named Eunice, who is seeking a divorce from her husband, Rudy. During this representation, Jennifer discovered some evidence that indicated that Rudy was sexually abusing Eunice's daughter. Eunice does not want Jennifer to report this to authorities, because she doesn't believe that it is true. Jennifer is not sure whether she has a duty to report the apparent child abuse to the state authorities pursuant to a state reporting statute. Nobody else in Jennifer's firm has any experience with family law. A friend of Jennifer, Isai, is an experienced domestic relations lawyer in a different law firm but is not her firm's ethics counsel. Jennifer has not talked with Eunice report the sexual about consulting another lawyer. May Jennifer consult Isai about whether he has a duty to report the sexual abuse? A. Yes, because there is nobody in Jennifers firm who is qualified to advise her. B. Yes, because the purpose of the disclosure is to obtain advice about whether jennifer must report the child abuse. C. No, because she has not obtained her client's informed consent. D. No, because Isai has not been designated as her firm's ethics counsel.

B. Yes, because the purpose of the disclosure is to obtain advice about whether jennifer must report the child abuse.

In the course of representing Frostco, Inc, Caroline receives a visit from Mark, a Frostco employee. She explains at the outset that she represents Frostco, not Mark. He says he understands that but needs to tell her about a problem off the record! Mark says that the Sales contracts that Caroline prepared for Frostco over a period of years included fraudulent misrepresentations about the durability of the refrigerators that Frostco was selling to distributors. Mark provides Caroline with secret internal memos in which company officers acklowledged that the refrigerators have substandard compressors that will last for only a tenth of the useful life stated in the contracts. These defects will cause substantial financial losses to the distributors, because the refrigerators will not work without functioning compressors, and many buyers will sue for breach of warranty. Caroline knows that Frostco probably will not be able to make good on the warranty claims because it does not have enough money to repair or replace all the refrigerators. Caroline confronts the general counsel of Frostco, who directs her to back off and continue preparing the contracts. He declines to address the product defect or to scale back the warranty claims in new contracts. He says, "If you don't want our business, we will find another lawyer who will write these contracts" Caroline decides to withdraw from representation of Frostco. May Caroline tell the distributors that her client misrepresented the durability of the refrigerators? A. Yes, because in addition to withdrawing from representation, Caroline is required to disclose the facts and circumstances relating to her client's fraud to the distributors. B. Yes, because the rules permit disclosure in these circumstances so that the distributors can try to recoup their losses. C. No, because Caroline did not assist Frostco in deciding how many years duration should be stated for the warranty on the refrigerators. D. No, because there is no indication that the misrepresentations create a risk of death or substantial bodily harm.

B. Yes, because the rules permit disclosure in these circumstances so that the distributors can try to recoup their losses.

Tandy is a practicing lawyer and adjunct law professor at Frostburg Law School, which is part of Frostburg University. Two Caucasian high school seniors have asked her to present them in their lawsuit against Frostburg University. They allege that Frostburg University did not admit them to the undergraduate program because of race discrimination. Specifically, they allege that students of color with similar test scores and grades were admitted to Frostburg University, while they were not. Tandy reasonably believes that she can competently and diligently represent the students in their lawsuit. She obtains informed consent, confirmed in writing, from both the students and the university. May Tandy represent the two students in their lawsuit? A. Yes, because the university gave informed consent, confirmed in writing. B. Yes, because the students gave informed consent, confirmed in writing. C. No, because the interests of the students are directly adverse to the interests of the university. D. No, because the university employs Tandy, so this representation would present a nonconsentable conflict of interest for Tandy.

B. Yes, because the students gave informed consent, confirmed in writing.

Attorney Ash is a solo practitioner specializing in serving small businesses. Ten years ago, Ash handled a routine health department license application for Sprinkles, a newly founded small family-owned bakery. Sprinkles is now known for its cupcakes. Death by Cupcake (DBC) is a bakery across town from Sprinkles that also specializes in cupcakes. DBC would like to hire Ash to represent them in negotiating a commercial lease to open a new bakery. As it happens, the property it wants to lease is across across the street from Sprinkles. The increased competition would significantly hurt Sprinkles business. May Ash accept the representation without obtaining Sprinkles' informed consent? A. Yes, because informed consent is not required when a prospective client and a former client are merely economic competitors. B. Yes, because the two matters are not substantially related. C. No, because BC and Sprinkles are economic competitors. D. No, because DBC's interests are materially adverse to Sprinkles.

B. Yes, because the two matters are not substantially related.

Larry, the father of Linda, Abby, and Carl, died at age 87. His property was to be divided equally among his children. Louise, Larry's partner, is executor of the estate. Most of his property is in cash and stocks, but Larry also left his children a lakeside cabin. The three adult children decide that Carl should take the cabin and should pay his sisters one-third of its value each out of his share of the cash and stocks. The appraised value of the cabin is $225,000, but Carl urges that, for the purpose of the transfer, the cabin should be valued at $150,000, because he believes the appraised value is unrealistic and that he will soon have to replace the water and septic systems for the cabin, at considerable cost. After some discussion, Linda and Abby are willing to go along with their brother's suggestion as to the valuation of the cabin because they care more about their brother than they do about the money. The three siblings need to hire legal assistance to prepare and file the documents relating to the transfer of the cabin. Carl proposes that they hire Morton, a lawyer he has used for other legal matters over the last ten years. Morton reasonably believes that he could competently represent all three siblings. He explains the potential problems and the advantages and risks of joint representation and obtains their consent to the joint representation. None of the siblings has any questions or new proposals, and he does not provide any legal advice to them. Morton then prepares a document valuing the cabin at $150,000 for purposes of the transfer, just as the siblings agreed. Each sibling confirms that all of them still want him to be their attorney. May he represent all three siblings in the transaction? A. Yes, because Morton merely prepared documents per their joint instructions, so he did not even need their informed consent. B. Yes, because there is no apparent conflict between the interests of the three siblings and he obtained their informed consent. C. No, because Carl would benefit by the cabin being assigned a lesser value, while Abby and Linda would be paid more if the assigned value is higher, so the conflict is non-consentable. D. No, because Morton's prior representation of Carl creates a significant risk that his representation of Linda and Abby would be materially limited by his felt loyalty to Carl.

B. Yes, because there is no apparent conflict between the interests of the three siblings and he obtained their informed consent.

Jaiden, an assistant general counsel of Plenum, Inc., a pharmaceutical manufacturer, was asked to investigate whether the research division of Plenum was concealing reports of adverse reactions to the company's best-selling product, Somalox, an anti-depressant. Two users of Somalox had contacted the general counsel's office, reporting that they had submitted adverse reaction reports to the research division, as directed on the package insert, but that they had received no response from the company. One of these two users alleged that he had suffered hallucinations and suicidal ideation after taking Somalox, and that he had described these reactions in his earlier report. The other user reported that she was hospitalized for depression after having taken Somalox for several weeks. She also had reported this to the research division. The non-response is worrisome. The company policy is to acknowledge receipt of any adverse reaction reports and to assure users that the company will report adverse reactions to the Food and Drug Administration as required by law. The research division would route any adverse reaction reports through the general counsel's office to the FDA, but none have been received or sent for Somalox. Jaiden schedules a meeting with Bertha, the chief of the research division. Jaiden shows her the correspondence sent to the general counsel's office by the two users and asks her to show him these and any other adverse reaction reports that the company has received. Bertha flushes and stammers in response to Jaiden's query. "I am afraid that I have let a few of these slip through the cracks. Since you are the company's lawyer, I can speak to you in confidence, right? Am I in trouble here?" In answering Berthas questions, which of the following statements would Jaiden be best advised to make? A. "Our conversation is protected by attorney-client privilege, so you can talk with me in confidence." B. "You can speak to me in confidence, because you are an employee of Plenum, and therefore your interests are aligned with those of the company.?' C. "I represent Plenum, so I cannot give you any legal advice except the advice to get a lawyer" D. "I have to report anything that you tell me to the company's leadership, because I represent the company, but I can assure you that the information won't be shared outside of the company - for example, with law enforcement agencies."

C. "I represent Plenum, so I cannot give you any legal advice except the advice to get a lawyer"

Pablo is a "notario," a non-lawyer who helps Spanish-speaking low-income undocumented immigrants with their immigration problems. He charges much less for legal advice than a lawyer would charge; in fact, his clients are so poor that they could not afford to pay any lawyer for the help that Pablo provides. He is committed to serving his community. One day, Pablo telephones Ann, an immigration lawyer he met at a party. Ann and Pablo are friendly because both of them serve truly indigent clients for no charge at all. Pablo says, "I've got a guy in my office, from Honduras, who wants to apply for asylum, but he has two misdemeanor convictions for possession of drug paraphernalia, for which he was sentenced, in each case, to a month in jail. He wants to know whether that is an absolute bar to getting asylum, in which case he would decide not to apply. What's the answer?" Ann has all the clients she wants at present and does not want to suggest that the man become her client. Which of the following responses is best? A. "It would not be an absolute bar, but it could affect an adjudicator's exercise of discretion, especially if he has been involved in any other misdeeds, such as domestic violence.? (That answer would be a correct statement of the law.) B. "I would like to help you, but I can't, because it is improper to give legal advice to an undocumented foreign national." C. "I would like to help you, but I can't, because it is improper for me to assist a notario in providing legal advice! D. "It's really important that you serve this man competently, so I'll give you the answer, provided that you call me with any other legal questions that may arise. But let's be clear about this: I am not going to be his lawyer; he's your responsibility, not mine."

C. "I would like to help you, but I can't, because it is improper for me to assist a notario in providing legal advice!

Assume that the lawyer's primary motive for each of the following is to earn money. Which one is prohibited by the Model Rules? A. A lawyer sees a post on Facebook from Pauline, whom he has never met, complaining that she had just discovered that she was a victim of identity theft. In a reply on Facebook to her post, the lawyer offers to represent Pauline in a civil suit for $150 per hour. B. After a lawyer learns that one of her former clients was the victim of medical malpractice, the lawyer visits the former client's home to tell him that she will represent him in a civil suit against the hospital and doctor for a one-third contingent fee. C. A lawyer is walking on the beach during a summer evening. He sees a drunk man throw his girlfriend against the wall, causing her to lose consciousness briefly. The boyfriend leaves the scene. When the woman comes to, the lawyer gives her his card and offers to represent her in a suit against the man on a contingent fee basis. D. Two days after a major gasoline company negligently causes an oil spill that results in severe property damage to several beachfront homes, a lawyer sends the owners of those homes a letter offering to represent them in a suit against the gas company. The letter states that he would charge them $250 per hour for his services.

C. A lawyer is walking on the beach during a summer evening. He sees a drunk man throw his girlfriend against the wall, causing her to lose consciousness briefly. The boyfriend leaves the scene. When the woman comes to, the lawyer gives her his card and offers to represent her in a suit against the man on a contingent fee basis.

Which one of the following statements would be protected by the attorney-client privilege if an adverse party sought to compel the lawyer to disclose the information? A. At a firm holiday party, a client with a products liability claim tells his lawyer's partner (who is not working on that case) that he plans to divorce his wife because she just wants his money. The partner informs the lawyer of that revelation. The wife's divorce lawyer later seeks disclosure of this conversation. B. A lawyer represents a doctor in a medical malpractice suit. The plaintiff is still in the hospital. To obtain facts about how much his client has been suffering, the lawyer interviews a patient in the bed adjacent to the plaintiff's bed. C. A paralegal interviews a client about his case, and she gives her notes to her supervising attorney. D. A lawyer advises several clients to buy certain securities that will minimize the clients' tax liabilities. The government believes that the securities are unlawful tax shelters and issues a subpoena to the lawyer, seeking the names of the clients.

C. A paralegal interviews a client about his case, and she gives her notes to her supervising attorney.

Attorney Leticia represents client Benjamin, a pharmacist, in an employment discrimination suit. With the help of his best friend Rocky, who happens to be sleeping on his couch that month, Benjamin writes a summary of the case, which Benjamin then emails to Leticia. Which one of the following statements is correct? A. Only Benjamin can be compelled to testify about the communication. B. Only Rocky can be compelled to testify about the communication. C. Benjamin or Rocky can be compelled to testify about the communication. D. Neither Benjamin nor Rocky can be compelled to testify about the communication.

C. Benjamin or Rocky can be compelled to testify about the communication.

Zarah is a partner at a law firm. She supervises the work of a junior associate, Charlie. They are representing Whitney in her divorce proceedings. The divorce involves a heated battle over the custody of the daughter of Whitney and her husband, Cody. Zarah asks Whitney whether she knows the password for her husband's e mall. Whitney gives it to her. Then Zarah gives the password to Charlie and instructs him to copy all the emails in cody's account and read them to see whether cody is having an affair. charlie objects. zarah says that this invesitgative work could win the case for Whitney and threatens to fire Charlie if he does not comply. charlie complies. they learn that cody really is having an affair and whitney wins custody. when cody discovers what happened he files a disciplinary complaint against zarah and charlie. assuming charlie's surreptitious review of cody's email is unlawful and a rule violation, which of the following statements is accurate? A. Only Charlie is subject to discipline. B. Only Zarah is subject to discipline. C. Both Charlie and Zarah are subject to discipline. D. Neither Charlie nor Zarah is subject to discipline.

C. Both Charlie and Zarah are subject to discipline.

Eduardo is a sole practitioner. He wants to handle the following matters and to make agreements with clients under which he would receive 20 percent of any recovery awarded to the client. In which of these cases would Eduardo be subject to discipline if he undertook the representation under those terms? A. Eduardo previously represented Atticus in a criminal matter. Atticus pleaded guilty and was incarcerated for a year. During that year, he reports, prison officials deprived him of medication that he needed, resulting in his having a mild stroke. Atticus wants to sue the prison for damages. B. Byron and his ex-wife are each half-owners of a mini-golf business. Byron has asked Eduardo to seek a partition (court-ordered division) of the property. C. Celeste has asked Eduardo to file an action seeking an order requiring her ex-boyfriend, who is the father of her daughter, to pay child support. D. In a fit of rage, Dana's former husband smashed her new sports car, requiring $12,000 worth of repairs. She wants to sue him for damages.

C. Celeste has asked Eduardo to file an action seeking an order requiring her ex-boyfriend, who is the father of her daughter, to pay child support.

Amar, a lawyer, has earned an undergraduate degree, a social work degree, a law degree, and an LL.M. degree. He now owes $260,000 in federal student loans, the last of which he obtained in 2016. He takes a job at his county's legal aid office. He enjoys the work there and can imagine working there, or in similar public service employment, indefinitely. But his salary is only $42,000 a year before taxes. He can't afford to make his monthly student loan payments on a standard ten-year repayment plan, or even on an extended 30-year repayment plan. He has an offer from a small law firm that would pay $54,000 a year, but the work would involve a lot of drudgery and be much less interesting than his legal aid job. What is his best option? A. Default on his loans and hope that the federal government does not sue him. B. Declare bankruptcy and hope that his loans will be discharged through that process. C. Elect to make monthly payments through the federal income-based repayment plan, even though this option will cause the amount he owes to rise rapidly, because he won't be paying all the interest that accrues each month. D. Quit his job and accept employment at the small law firm.

C. Elect to make monthly payments through the federal income-based repayment plan, even though this option will cause the amount he owes to rise rapidly, because he won't be paying all the interest that accrues each month.

On a dark night in a bad part of town, a young man named Alan is found dead. Chad is arrested and put in jail, accused of murdering Alan. Chad tells Alberto, his lawyer, that he could not have murdered Alan, because at the time of the murder, he was burying the body of another man, Ethan, whom he had killed. Ethan has been declared missing, and the police are investigating whether foul play has occurred. Chad tells Alberto where he buried Ethan. Obviously, Chad does not want anyone to know his alibi because then he would be charged with another murder. In exchange for the alibi information, Alberto could obtain a desirable plea bargain for Chad on the current and prospective charges. May Alberto reveal this information to the prosecutor without consulting Chad? A. Yes, because he can use the information to obtain a desirable plea bargain for Chad on the current and prospective charges. B. Yes, because unless he reveals the information, he is impeding the state's investigation of Ethan's disappearance. C. No, because Alberto obtained this information in the course of representing Chad. D. No, because Alberto does not have objective evidence indicating that Chad killed or buried Ethan.

C. No, because Alberto obtained this information in the course of representing Chad.

Salima represents Anthony, a man accused of stealing money from his company. Anthony was a bookkeeper at a small bakery. His duties included administering the payroll system. Anthony altered a computer program so that he would be paid an extra few hundred dollars a month by the company. He got this higher pay every month for several years. Anthony's theft from the company was noticed during an accounting audit and he is facing criminal charges. During a meeting to discuss the criminal charges, Anthony tells Salima that he also rigged the program so that he would also receive approximately $200/month that should have been paid to Jason, a custodian with six children. He says that after he was caught stealing, Anthony re-programmed the system so that the checks would be properly paid going forward. Neither Jason nor the company are aware that Anthony stole from Jason. Salima asked Anthony if he plans to fess up and repay Jason. He says "No way. I'm in enough trouble already." Salima disapproves but says nothing. Salima meets with the company's general counsel to discuss restitution to the company. The general counsel tells Salima that the audit is complete and presents an accounting that lists only the money that Anthony stole from the company. Salima reasonably believes that unless she reveals the theft, there will be no restitution to Jason. May Salima reveal this information without Anthony's consent? A. Yes, because revelation is necessary to ensure that Anthony will be asked to repay Jason. B. Yes, because in stealing Jason's money, Anthony committed a crime. C. No, because Anthony did not use Salimas services to steal Jason's money. D. No, because the company's general counsel has stated that the audit is complete.

C. No, because Anthony did not use Salimas services to steal Jason's money.

Attorney Dahlia represents restaurant owner Stefan in a negligence suit. During the course of her representation. Dahlia learns that Stefan uses PestOFF, a rat poison, in his kitchen PestOFf has recently been shown to cause birth defects if consumed, even in tiny amounts, by pregnant women, and its use within the state has been banned. Its use is a misdemeans: punishable by imprisonment for up to six months. The product was applied only once, ant only to the floor of a closet where cleaning supplies were kept. It was never used near food. There is nothing to suggest any ingestion or injury to a person A city ordinance requires anyone, including lawyers and doctors, who learns of the use of PestOFF to report it to local police immediately. Failure to report is a violation punishable by a $50 fine. Dahlia confronts Stefan and insists that he stop using this product and that they report his past use to the police. Stefan agrees to stop using the product but refuses to report his past use to the police, Dahlia then withdraws from representation of Stefan and reports Stefan's use of PestOFF to the police. Is Dahlia subject to discipline for reporting Stefans use of PestOP to the police? A. Yes, because she knew of no potential harm to a person from the use of PestOFF. B. Yes, because her duty to protect Stefans confidences continued even after she terminated her representation. C. No, because Dahlia's disclosure is permitted under these circumstances. D. No, because by using PestOFF, Stefan had committed a crime.

C. No, because Dahlia's disclosure is permitted under these circumstances.

Ian, a lawyer, is trying to build up his client base. He frequently reads in his local newspaper about automobile accidents in which pedestrians are injured. He would like to visit the victims, either at hospitals or their homes, within a few days after the accidents, before insurance adjusters contact those potential plaintiffs and persuade them to sign settlements in which they receive only small amounts of compensation. He would represent these clients on a contingent-fee basis. Ian knows that the ethics code imposes some restrictions on solicitation, but he thinks he has a right to reach out to these victims before the insurance adjusters get to them. Assuming that none of the victims are people known to Ian or his partners, and that none of them are lawyers, may Ian visit the victims as he desires? A. Yes, because the U.S. Constitution protects his right to free speech and association. B. Yes, because the insurance adjusters are allowed to contact the victims. C. No, because Ian is pursuing this work to acquire fee-generating cases. D. No, because if he wants to challenge the validity of any ethics rule, he may only petition the court that issued it or bring an affirmative lawsuit to challenge the rule.

C. No, because Ian is pursuing this work to acquire fee-generating cases.

Attorney Priya represents Janice in her suit against major canned goods company, after she contracted botulism from a can of pickled fish. Priya receives an email from opposing counsel saying that he is ready to meet to discuss settlement. Priya asks Janice if they can meet the next day to talk about what terms Janice wants. When they meet, Priya tells Janice that she thinks she could get at least $4 million dollars in a settlement. Janice is thrilled at that number, and she tells Priya, "I didn't think we could get nearly that much! You have been such an amazing lawyer and friend throughout this process. If they come to us with an amount lower than that, and you think that we can get more by rejecting the offer, I trust your judgment. Do what you think is best." The next day, Priya meets with opposing counsel, who informs her that the company is offering $3.25 million dollars to settle the case. Without conveying the offer to Janice, Priya responds, "That number does not come close to addressing the pain and suffering my client has been through. My client rejects your client's offer." The case goes to trial, and Janice loses and recovers nothing. She complains to the bar. Is Priya subject to discipline for not telling Janice about the offer? A. Yes, because a lawyer must communicate a settlement offer to a client, and Priya did not do so. B. Yes, because a lawyer must promptly inform the client of any decision or circumstance with respect to which the client's informed consent is required, and Priya did not do so. C. No, because Janice authorized Priya to reject the settlement offer. D. No, because the settlement offer was less than the amount that Janice deserved, so Priya made a reasonable judgment in rejecting it.

C. No, because Janice authorized Priya to reject the settlement offer.

Eleanor represents Carmichael Inc., an energy company that sells electricity to residential and commercial customers. The corporation has five stockholders; four are Carmichael family members, and the fifth is an outside investor who provided start-up funds five years ago. In the course of her review of corporate documents, including e-mails exchanged among company personnel, Eleanor discovers that Marcus, the company's Director of Information Technology, has been harvesting cell phone numbers from employees and customers' e-mail messages that went through the company's server and selling them to telemarketing companies. This information about Marcus's activities, if it became public, would be very damaging to Carmichael In's reputation. She consults Carmichael's president, Joseph Carmichael, who says that he has suspected that something like this might be happening, but that Eleanor should just leave it alone. Eleanor suspects that Joseph may be collaborating with Marcus and sharing the revenue from the telemarketing companies. Eleanor goes to a meeting of the board of directors that is attended only by Joseph and his wife and son, who are also members of the board, and Joseph again tells her to leave it alone. His wife and son concur. The company is regulated by the Public Service Commission. Eleanor continues to believe that exposure would hurt the company's reputation and therefore its future profits. Do the Rules require Eleanor to reveal the misconduct to the commission? A. Yes, because what Marcus has been doing is harmful to members of the public. B. Yes, because Rule 1.13 requires such disclosure. C. No, because Rule 1.13 does not require such disclosure, and the disclosure would violate Rule 1.6. D. No, because although Rule 1.13 would require such disclosure, Rule 1.6 prohibits it.

C. No, because Rule 1.13 does not require such disclosure, and the disclosure would violate Rule 1.6.

Attorney Suzette is a solo practitioner in a small town. She specializes in medical malpractice cases and has handled hundreds of such cases over the past two decades. Suzette's friend Gunter hosts a local radio program, and he offers Suzette a 15-second advertising spot in his program. Suzette's ad reads, in its entirety: "Have you or has someone you know been hurt by a doctor's negligence? Attorney Suzette Bellows can help. Bellows specializes in medical malpractice cases. Call (555) DOC-HELP now for a free consultation." Gunter plays a recording of the ad on the radio. Is Suzette subject to discipline? A. Yes, because the ad states that she specializes in medical malpractice claims. B. Yes, because the ad did not contain her office address. C. No, because any regulation of lawyer advertising violates the First Amendment. D. No, because everything in the ad is truthful.

D. No, because everything in the ad is truthful.

Attorney Uma is a solo practitioner who represents Royce in a civil case. She is scheduled to argue an important motion at a hearing three days from now. She learns that her father has passed away, and his funeral will take place on the same day as the hearing. The judge has denied her motion for a continuance, because she has previously been granted rote postponements of the argument of this motion. Uma's good friend Wendell is a seasoned litigator. She reasonably believes that he can competently represent Royce at the hearing, so she asks him if he can handle the hearing for her, and he agrees. He agrees to treat the file as confidential and to return it to Uma promptly after the hearing. Uma leaves a phone message for Royce, but he doesnt return her call. She gives Wendell Royces file so that he can familiarize himself with the case. Assume that Wendell can competently represent the client at the hearing. Was it proper for Uma to give Wendell the file? A. Yes, because Uma reasonably believes that Wendell can competently represent the client at the hearing. B. Yes, because Wendell agreed to treat the file as confidential and to return it to Uma promptly after the hearing. C. No, because Uma did not obtain Royces prior consent. D. No, because Uma has not filed a motion to withdraw from representation of Royce.

C. No, because Uma did not obtain Royces prior consent.

Angus hired Vojtech to file a lawsuit against Linkbook, a major social media outlet. He claimed that when Byron, the founder of the company, was just starting Linkbook, Angus made a small personal investment in the company ($1,000) and that in exchange for his investment, Byron promised that would receive 20 percent of the earnings of Linkbook. After Vojtech did some investigation, he obtained indisputable factual evidence that Angus' claim was entirely fraudulent. Angus then admitted to Vojtech that his claim was baseless but asked Vojtech to press the claim anyway, because he thought that Linkbook would pay him something just to end the lawsuit. Vojtech declined to file the lawsuit and withdrew from representing Angus. Angus then hired a new lawyer, who filed a lawsuit against Byron and Linkbook. Vojtech has not informed either the lawyer who is now handling the suit or the judge who will try the case about Angus's fraudulent claim, and he will not do so even if Angus gets a handsome settlement. Is he subject to discipline? A. Yes, because a lawyer may not assist a client or a former client to commit a crime or fraud. B. Yes, because Angus's fraudulent conduct is related to a judicial proceeding. C. No, because Vojtech is obliged to protect Angus's confidences. D. No, because Angus' fraud is not reasonably certain to cause substantial injury to the defendants' property.

C. No, because Vojtech is obliged to protect Angus's confidences.

Attorney Yoshi represented client Cyrus in his divorce. During the course of the representation, Cyrus told Yoshi that if his ex-wife, Tisha, was awarded their beachfront vacation home in Florida, he would burn it down when it is vacant, because he could not stand the thought of his ex-wife using the vacation home without him. Yoshi advises Cyrus that this is a terrible idea. He points out that deliberate arson could lead to criminal charges and civil liability, not to mention termination of visitation rights. He also cautions that Cyrus could injure or kill a member of his family if he burns down that house. Cyrus repeats to Yoshi that he would not do this if anyone was in the house, but he does not assure Yoshi that he won't torch the building. The divorce was finalized, and Tisha was awarded the vacation home. Yoshi has terminated his lawyer-client relationship with Cyrus. Yoshi reasonably believes that revelation is necessary to prevent Cyrus from committing a crime that is reasonably certain to result in substantial injury to Tishas house. May Yoshi warn Tisha or her attorney that Cyrus is planning to burn down the vacation home? A. Yes, because Cyrus is no longer Yoshi's client. B. Yes, because Yoshi reasonably believes that revelation is necessary to stop Cyrus from burning down the house and committing the crime of arson. C. No, because Yoshi learned this information during the course of the representation. D. No, because the rules bar lawyers from revealing their clients' stated intentions to destroy the property of others.

C. No, because Yoshi learned this information during the course of the representation.

Margery owns and manages a successful law practice. Her specialty is representing plaintiffs in employment discrimination cases alleging gender discrimination. Recently, she has gotten referrals of a substantial number of cases involving people who have suffered sexual harassment or sexual assault in the workplace. At the end of every calendar year, Margery directs her office manager to send a $50 bottle of champagne to every individual who referred a new client to her practice that year to thank them for sending clients her way. The recipients include other lawyers, clients, former clients, and friends, relatives, or any others who have referred matters to her. Giles, an estate planning lawyer, referred a prospective client to Margery whom she represented in an administrative matter. Giles, who does not drink alcohol, received a bottle of champagne from Margery. Giles files a complaint against Margery with the local bar counsel for giving gifts to get business. Is Margery subject to discipline? A. Yes, because a lawyer may not give an inducement to another person to refer a case to the lawyer. B. Yes, because the recipient of a bottle of champagne might perceive it as compensation for the referral. C. No, because a $50 bottle of champagne is not greater in value than a token item that might be given for the holidays or for other ordinary social hospitality. D. No, because a lawyer may give any tangible thank-you gift to another lawyer who refers a matter as a professional courtesy.

C. No, because a $50 bottle of champagne is not greater in value than a token item that might be given for the holidays or for other ordinary social hospitality.

In answering this question, use the facts of the previous question and assume that Morton has properly undertaken the representation of the three siblings. In the course of his research before preparing the property transfer documents for Larry's summer cabin, Morton discovers that Carl could obtain a reduction in tax liability for the cabin if the transfer of the cabin is postponed until next year. Morton knows that Linda and Abby are eager to conclude the transaction and that they will be able to earn more interest on funds paid to them by Carl if the payment is made this year instead of next year. Carl asks Morton whether he could postpone the transfer of the cabin until next year, slowing things down without revealing any reasons for the delay. May Morton delay the transaction without disclosing the reasons to Carl's sisters? A. Yes, because the adverse consequence to them from the delay will not be as significant as the resulting benefit to Carl. B. Yes, because this will allow him to avoid the clients' potentially adverse interests and to focus on their common interests. C. No, because each client is entitled to be told information relating to the representation that might affect the client's interests. D. No, because Carl's suggestion would involve Morton in a fraud against Linda and Abby, requiring Morton's immediate withdrawal from representation of Carl.

C. No, because each client is entitled to be told information relating to the representation that might affect the client's interests.

Spencer desires to represent his golfing partner Craig, who was injured on a ski slope that was apparently negligently maintained. Spencer tells Craig on the phone that he is willing to work on a contingent fee basis, under which Craig will be charged nothing unless Spencer obtains a settlement or wins a judgment. In either case, Spencer would charge 33 percent of any recovery after the deduction of the expenses of litigation. Spencer explains that Craig will not be responsible for any litigation expenses unless there is a recovery that exceeds the amount of the expenses. They discuss these points, and Craig agrees to the fee arrangement. Spencer contacts the ski slope's insurer, and within three weeks, with Craig's approval, the case is settled for $21,000. Spencer transmits $14,000 of that settlement to Craig. Spencer spent only four hours on the case. Was Spencer's conduct proper? A. Yes, because he disclosed all the details of the fee arrangement. B. Yes, because a lawyer's rate need not be disclosed in writing. C. No, because he did not disclose all the details of the fee arrangement in a writing signed by Craig. D. No, because a rate of 33 percent is unreasonable for a matter that required so little of the lawyer's time and attention.

C. No, because he did not disclose all the details of the fee arrangement in a writing signed by Craig.

Burke is a trusts and estates lawyer. Ida, an acquaintance, asks him to represent her in a negligence lawsuit against a major retail company. she stands to make millions from the lawsuit, and she is happy to hire Burke on a contingent fee basis. Burke has never before handled a negligence action and he has never handled a trial of any kind. Burke accepts the case. He does not tell Ida that he has no experience in handling cases of this sort whom he could consult, but unfortunately he does not have such contact. He studies the relevant law and procedure. He performs well but loses the case. Is Burke subject to discipline? A. Yes. because he did not tell Ida that he had never handled a negligence claim or a trial B. Yes, because he failed to associate with a lawyer experienced in handling negligence actions. C. No, because he studied the relevant law and procedure and performed well D. No, because he is licensed to practice law in the state and may therefore accept any litigated matter in the state.

C. No, because he studied the relevant law and procedure and performed well

Tomas, an attorney, represented the IWT Corporation, which fired a woman named Celinda. IT claims that Celindas written work was sloppy; Celinda claimed that IWI simply wanted to give the job to a younger person. She wanted her job back or a substantial cash settlement. Pursuant to a clause in Celindas employment contract, her dispute had to be resolved through arbitration. While the arbitration proceeding was pending, IWT's general counsel read the emails that Celinda had sent to Tomas through the company's email system, from the time that Celinda received notice that she was being fired until a month later, when she actually departed. IT's 250- page office handbook, a copy of which was given to Celinda two years ago, says on page 138 that the e-mail system belongs to the company and that information sent over the system may be read or disclosed by company officials. One of Celindas e-mails said, "I know that my work isn't as good as it used to be, and that I have been making a lot of mistakes this year. So maybe they do have good reason to fire me after all.* IWT's general counsel sent a copy of this e-mail to Tomas. Tomas did not tell Celindas lawyer that he had a copy of the email, planning to use it to surprise her during the arbitration proceeding. Is Tomas subject to discipline? A. Yes, because this document consisted of electronically stored information relating to the representation and he therefore had a duty to notify Celinda, the sender, or her attorney. B. Yes, because Celinda's communications with her lawyer were privileged. C. No, because no ethics rule requires this disclosure, even if litigation or arbitration is pending. D. No, because the matter is in arbitration, and the disclosure would be required if litigation were pending.

C. No, because no ethics rule requires this disclosure, even if litigation or arbitration is pending.

In answering this question, use the facts of the previous question. Must Eleanor withdraw from representation of Carmichael? A. Yes, because a lawyer must withdraw from representation of an organizational client when she discovers that an employee of the client corporation has committed misconduct. B. Yes, because the board of directors failed to address the misconduct after Eleanor called it to the board's attention. C. No, because nothing in Rule 1.16 requires withdrawal. D. No, because she represents the corporation, not Marcus.

C. No, because nothing in Rule 1.16 requires withdrawal.

Evelyn, a recently admitted attorney, is serving as a law clerk to Judge Leopold Osterman on the state court of appeals. The judge has heard the appeal in a contract dispute in which IGL Corp. has sued Mountain Hardware, Inc., for damages. Mountain Hardware was rep resented by the law firm of Westerfield & Pilson, which Evelyn worked for during her second summer in law school. She did not work on that dispute while she was at the firm, because the firm accepted it after she left the job, but Judge Osterman has asked her to write the opinion in the case. While she was working on the opinion, Margaret Pilson from Westerfield & Pilson emailed Evelyn, on her private email account, asking whether she would like to discuss joining the firm as an associate after she finished her clerkship. Evelyn wanted to take that job, so she told Judge Osterman about the email, and he said that it was fine to discuss a possible job with the firm. Evelyn and the firm then agreed on a start ing salary of $95,000, and that she would begin work two weeks after her clerkship ended. A week later, Evelyn completed the opinion, which the judge adopted. It dismissed the case against Mountain Hardware. Is Evelyn subject to discipline? A. Yes, because she entered into negotiations and a future employment agreement with a firm while participating personally and substantially as a judicial law clerk writing an opinion that would affect a present client of that firm. B. Yes, because her opinion favored the firm that was offering her a job. C. No, because she notified her employer of the overture from the law firm and her intent to pursue it. D. No, because she did not submit the opinion to the judge until after she had accepted the firm's offer, so she could not have been helping a client of the firm in order to procure a job offer.

C. No, because she notified her employer of the overture from the law firm and her intent to pursue it.

Paul, an associate justice of the United States Supreme Court owns a substantial amount of stock in a publicly traded corporation called Exrix, LIC. The Court grants certiorari to decide a class action suit brought by shareholders of Exrix. Paul has disclosed his stock ownership but is not planning to sell his stock. No party has made a recusal motion. Must Paul recuse himself pursuant to a code of judicial conduct? A. Yes, because he owns some stock in a corporation whose case he must decide. B. Yes, because the amount of stock that he owns is substantial. C. No, because no party has made a recusal motion. D. No, because no ethics rule requires him to recuse himself.

D. No, because no ethics rule requires him to recuse himself.

Esteban, a lawyer in State A with a general practice, regularly represents and advises Patrick who lives just a few miles away in State B. Patrick owns and operates a hunting supply store in State A. For years, Esteban has advised him about employment and tax issues related to his business, Patrick has amassed enough capital to start a second business, a wilderness tour company, and he decided to locate this enterprise near his home in State B. He asked Esteban if he would meet with him once at the new office to advise him on State B tax and Ital estate law to assist him in launching the new business. Esteban has the knowledge to Provide competent legal advice to Patrick on these issues even though he is not a member of the bar of State B. He would not charge Patrick fees that were higher than those permitted by State B. Esteban has not mentioned to Patrick that he is not licensed in State B; it did not cross his mind. Would Esteban be subject to discipline if he provides legal advice to Patrick as Patrick requests? A. Yes, because he has not disclosed to Patrick that he is not licensed to practice law in State B. B. Yes, because he is not licensed to practice law in State B. C. No, because the advice arises out of Esteban's practice in State A. D. No, because Esteban is licensed to practice law and could competently provide the advice.

C. No, because the advice arises out of Esteban's practice in State A.

Attorney Salim represents Roland, a doctor, in a medical malpractice case. In an e-mail, Roland reveals to Salim that on the day in question, he had had a couple of drinks before he performed the surgery at issue. He asks Salim whether that would expose him to punitive damages. Salim replies that it would not under the law of the state. Roland is to testify at the trial. In preparing for his testimony, Roland asks Salim whether, if the opposing counsel asks Roland whether he had anything to drink on the day in question, he can successfully invoke attorney-client privilege to avoid answering the question. Salim's answer should be: A. Yes, because the information is protected by attorney-client privilege. B. Yes, because the e-mail exchange is protected by attorney-client privilege. C. No, because the attorney-client privilege does not shield him from having to answer the question. D. No, because although Roland's statement to Salim that he was drinking would have been privileged if it had been made orally, it is not covered by attorney-client privilege because it was transmitted via e-mail.

C. No, because the attorney-client privilege does not shield him from having to answer the question.

In a state in which judges are elected, Damian Garner, the President of Garner Industries, a manufacturer of firearms, has donated $3,000 to the successful campaign of Ambrose Zoltan, who was elected as a judge of the state's highest court. That contribution represented 2 percent of the campaign contributions that the judge received. All contributions were disclosed pursuant to the state's campaign finance disclosure law. A gun control group sued Garner Industries for violating the state's gun control laws by manufacturing and selling guns with built-in silencers. The court ruled against Garner and it has appealed to the state supreme court, arguing that the gun control law is inconsistent with the Second Amendment. The gun control group filed a motion asking Judge Zoltan to recuse himself, but he denied the motion. Would Judge Zoltan's participation in the adjudication of this case be unconstitutional? A. Yes, because Garner made a substantial contribution to his campaign. B. Yes, because not all parties have consented to his participation. C. No, because the contribution was only a small part of his campaign treasury. D. No, because the contribution was disclosed.

C. No, because the contribution was only a small part of his campaign treasury.

Attorney Charlie works at a law firm of 30 lawyers in a large city. He represents his friend Suma in a wrongful discharge suit against her former employer, an architecture firm, Suma and her husband, Todd, are in the process of divorcing. Suma has retained a family lawyer associated with another firm to represent her in the divorce case. Charlie just learned that one of his law partners, Angus, is representing Todd in the divorce Charlie and Angus agreed not to communicate with one another about either of the matters. They did not speak to Suma and Todd about the conflict. May Charlie continue to represent Suma in her civil suit against her former employer? A. Yes, because Charlie and Angus agreed not to communicate with one another about either of the matters. B. Yes, because the conflict involves only Charlie's personal interests and does not present a significant risk of materially limiting Angus's representation of Todd. C. No, because the firm did not obtain both clients' informed consent, confirmed in writing. D. No, because law firms are not permitted to sue current clients.

C. No, because the firm did not obtain both clients' informed consent, confirmed in writing.

Attorney Damon represents his friend George in a civil case in which George is a defendant. George is alleged to have driven into the plaintiff's fence negligently. George and Damon meet in Damon's office to discuss the case. After some discussion about their children, who play together, George tells Damon that he was driving under the influence of PCP at the time of the incident. Several months later, George's wife files for divorce, and her lawyer subpoenas Damon to testify. The wife's attorney wants Damon to testify that George told Damon that he drove under the influence of drugs. May Damon be required to testify about George's use of PCP? A. Yes, because a subpoena has been issued. B. Yes, because the litigation in which Damon represented George was unrelated to the present litigation in which Damon will testify. C. No, because the information is privileged. D. No, because George shared this information with Damon before Georges wife filed for divorce.

C. No, because the information is privileged.

Troy goes to see Samantha, an attorney, to see if she can help him with a legal problem. Troy worked as a salesperson for three years for Glory Vacations, Inc., which sells vacation timeshares. Heidi, one of Glory's customers, has sued both Glory and Troy for $50,000, alleging that Troy made fraudulent statements while selling her some timeshare property. Upon receiving the summons, Glory fired Troy and cross-claimed against Troy, claiming that any fraud was unauthorized and is Troy's fault. Troy says he is being scapegoated and never made any false statements. Troy wants Samantha to represent him. He is now unemployed and has very little money. Samantha cannot afford to accept him as a pro bono client. Troy says that he has been turned down by several other lawyers because of his financial situation. He owns a house, so the local legal services office will not represent him. Without representation, Troy, whose formal education ended after high school, will be unable to defend himself effectively. He asks Samantha whether he has a due process right to have the court appoint a lawyer to represent him without charge. What should Samantha say? A. "Yes, because you are an indigent defendant." B. "Yes, because you can't afford a lawyer." C. "No, unless you sell your house and invest the proceeds, and your income remains below the federal poverty level.? D. "No, even if your house is repossessed and you become totally indigent."

D. "No, even if your house is repossessed and you become totally indigent."

Five years ago, Tammy was murdered at night while alone in her home. Her boyfriend Ray, with whom she had been quarreling, was accused of the crime. At first he claimed innocence and said she must have been killed by an intruding stranger. Initially, he declined the opportunity to consult a lawyer. After 12 hours of police questioning, he confessed to the crime. Subsequently, he retained Ming, a lawyer. Ray was convicted, largely on the basis of his confession, and was sentenced to 40 years in prison. Ray is now in prison. Daniel, a prosecutor in the county where Ray was tried, recently negotiated a plea bargain with the attorney for Burt, another man who was alleged to have committed several murders. Burt agreed to plead guilty to manslaughter and to receive a life sentence to avoid being charged with a capital offense. Part of the deal is that he would provide Daniel with details on his other crimes. During those discussions, Burt revealed that he had robbed and killed Tammy. He had dated her a year earlier, but she had rejected him in favor of Ray. Burt consents to provide a sample of his blood, which matches the blood that was found in Tammy's house the day after the murder. Ray's blood had not been found in the house. What if anything must Daniel do? A. Nothing, because Ray confessed, so he would have been convicted anyway. B. Promptly disclose this new evidence of Ray's innocence to Ming (unless a judge orders a delay in disclosure), but not to the court or chief prosecutor. C. Promptly disclose this evidence to Ming (unless a judge orders a delay in disclosure), and also disclose the new evidence to a court or to the chief prosecutor. D. Make a motion to the court to reopen Ray's case.

C. Promptly disclose this evidence to Ming (unless a judge orders a delay in disclosure), and also disclose the new evidence to a court or to the chief prosecutor.

The police arrested a middle school band teacher for repeatedly sexually abusing numerous female students aged 9 to 12. The molestation occurred during private music lessons and took place over a seven-year period before the teacher was arrested. Some of the victims told the police that they had reported the abuse to the school principal after it occurred, but the principal had failed to take action against the teacher. After the teacher was arrested, some of the victims and their families filed suit against the principal and the school board. The school board hired the firm of Schoenholtz & Koplow "to investigate the response of the school administration to allegations of sexual abuse of students." The board explained that it would need a full report so that it could decide whether to settle or litigate the claims against it. Meanwhile, the school board also hired a respected trial lawyer, Lindsay, at another firm, to represent the board in the lawsuit. Schoenholtz & Koplow interviewed all the officers of the school board to find out what they knew about the molestation, when they learned what they knew, and what they did after they received the information. Their notes and report were turned over to the school board, which gave them to Lindsay. The plaintiffs in the lawsuit requested, during discovery, the notes made by Schoenholtz & Koplow during their interviews. Lindsay must: A. Turn over the notes to the plaintiffs because while the communication may be protected, the underlying facts are not protected from disclosure. B. Turn over the notes to the plaintiffs because Schoenholtz & Koplow was hired to do the investigation, not to defend the school board against the lawsuit. C. Refuse to turn over the notes to the plaintiffs because they are protected by attorney-client privilege. D. Refuse to turn over the notes to the plaintiffs, based on attorney-client privilege, because the crimes discussed in those documents could result in liability for the school board.

C. Refuse to turn over the notes to the plaintiffs because they are protected by attorney-client privilege.

Which one of the following statements is correct? A. Congress and the state legislatures may not adopt binding rules of conduct for lawyers. B. The American Bar Association is the principal regulator of lawyers in the United States. C. State and federal courts adopt ethical rules that govern lawyers admitted to practice before them. D. State supreme courts have the exclusive authority to regulate the lawyers who practice in each state.

C. State and federal courts adopt ethical rules that govern lawyers admitted to practice before them.

The state in which Stella practices recently enacted an animal cruelty law. Stella represents Chix, Inc., a poultry company, in a suit brought by a local animal rights group. The suit alleges that the poultry company has violated the new state law by failing to provide adequate ventilation in the chicken coops. The suit seeks punitive damages. Stella files a motion to strike the punitive damages request, urging that the new law does not allow recovery of punitive damages. In an unrelated matter, Stella represents a group of students at a small Christian college, seeking to recover punitive damages against the school for allowing dissection of live animals in their school's biology classes in violation of the statute. When Stella agreed to represent the students, she explained to them that she was arguing against the availability of punitive damages in the Chix case. Stella is representing the students pro bono, so even if punitive damages were granted, she would not receive a portion of the award as a legal fee. Which of the following factors is least relevant to determining the risk that Stellas actions on behalf of one client would materially limit Stellas effectiveness in representing the other? A. The fact that she will represent both clients in lawsuits at the trial court level. B. The fact that the lawsuits will be litigated during the same period of time. C. The fact that Stella would not benefit financially if the statute were interpreted to allow punitive damages. D. The fact that, when Stella agreed to represent the students, she explained to them that she was arguing against the availability of punitive damages in the Chix case.

C. The fact that Stella would not benefit financially if the statute were interpreted to allow punitive damages.

Libby, a lawyer, represents Lawncare Enterprises, the defendant in a negligence case. Lawncare Enterprises manufactures power lawn mowers. Pierre, the plaintiff, had purchased a Model LC-15 Lawncare mower. One day when Pierre was operating the mower, the mower hit a small stone, which flew up and hit Pierre's left eye, blinding that eye permanently. Pierre claims that Lawncare knew or should have known that the protective shield on the mower he used was too short to prevent an accident of this type. During preparation for trial, Libby asked Ira, Lawncare's chief of customer relations, whether his department had received other complaints of stones or other small objects being thrown up during use of the Model LC-15. Ira said that the department had received more than two dozen such complaints, about half of which had involved minor injuries, but no serious injuries. During the trial, Ira was asked on cross-examination whether his department had received any prior complaints of injuries resulting from the use of the Model LC-15. Ira replied that there had been two or three complaints of scratches from debris kicked up by the mower, but none of those were serious, and none of those incidents had resulted in litigation. Libby observed this testimony by Ira without betraying, through her expressions or otherwise, that it was inconsistent with what Ira had told her earlier about the number of injuries that had been reported. A jury ruled in favor of Lawncare, and Pierre appealed. While she is working on the appeal, Libby visits the Lawncare factory and runs into Ira. She asks him why he had told her that about half of the two dozen complaints about the LC-15 resulted in injuries but then testified that there had been only two or three complaints of scratches. Ira replies that Lawncare's general counsel had advised him to testify truthfully but to try to minimize how much Lawncare knew about prior injuries caused by the product. He added that he had been careful to say that there had been two or three complaints of scratches, without saying that those had been the only complaints of injuries. What if anything should Libby do as a result of this disclosure? A. Nothing, because misleading testimony by a witness is not perjury unless that witness has made a literally false statement. B. Nothing, because the trial is over. C. Contact senior management officials at Lawncare and advise them to inform the trial judge that Iras statement at the trial was not true - but do nothing further if they choose not to so inform the judge. D. Contact senior management officials at Lawncare and advise them that if they do not inform the trial judge that Iras statement at the trial was not true, she will be obligated to do so.

D. Contact senior management officials at Lawncare and advise them that if they do not inform the trial judge that Iras statement at the trial was not true, she will be obligated to do so.

During his final year of college, Henry got excellent grades and achieved a very high LSAT score. However, during that year, he had two car accidents, ran up $11,500 in credit card debt, and was fired from his part-time restaurant job after he inexplicably failed to show up for work for ten days. Henry was then evaluated and diagnosed by a psychiatrist, who concluded that Henry suffered from bipolar disorder. This condition, if untreated, can cause extreme shifts in mood, energy, and ability to function. Henry's doctor explained that what he went through during his senior year was a manic period. The doctor prescribed medication. Ever since, Henry has been taking his medication as prescribed. Henry's doctor considers his illness "well-managed," and has told Henry that it would not affect his ability to practice law. Henry started law school three months after he finished college. Now he has completed law school, graduating at the top of his class. He is well-liked by his peers. During his third year, Henry enrolled in a clinical course. He won both of his cases and he received an A in the course. Henry is applying for admission to the bar. The character and fitness questionnaire asks, among other questions: 1) Within the past five years, have you exhibited any conduct or behavior that could call into question your ability to practice law in a competent, ethical, and professional manner? If you answered "yes," furnish a thorough explanation below. 2) Do you currently have any condition or impairment (including, but not limited to, substance abuse, alcohol abuse, or a mental or emotional or nervous disorder or condition) that in any way affects your ability to practice law in a competent, ethical and professional manner? If your answer is "yes," are the limitations caused by your condition or impairment reduced or ameliorated because you receive ongoing treatment? Henry believes that his disorder does not affect his ability to practice law in a competent, ethical and professional manner, and his physician agrees and has supplied him with a statement. Which one of the following statements is the most accurate? A. Henry need not disclose his condition, because federal disability law prohibits discrimination on the basis of physical or mental disability. B. Henry need not disclose his condition because he believes that his disorder does not affect his ability to practice law in a competent, ethical, and professional manner. C. Henry need not disclose his condition because his physician has concluded that his illness does not affect his ability to practice law in a competent, ethical, and professional manner. D. Henry must disclose the diagnosis and provide detailed information about it.

D. Henry must disclose the diagnosis and provide detailed information about it.

Armand, a lawyer, represented Walter, a plastic surgeon, during Walter's contested divorce from his wife five years ago. Armand no longer has any contact with Walter, and he does not remember anything about Walter's case. Recently, one of Walter's patients, Celia, developed serious complications and nearly died as the result of the extensive cosmetic surgery that Walter performed on her. She wants to sue him for punitive damages based on gross negligence. She has asked Armand to represent her. May he do so without Walter's consent? A. Yes, because he no longer represents Walter and the medical malpractice matter is unrelated to the divorce. B. Yes, because he does not remember anything about Walter's case. C. No, because a lawyer may not sue a former client without the former client's consent. D. No, because Armand would normally have obtained information in the divorce case that could be helpful to Celia.

D. No, because Armand would normally have obtained information in the divorce case that could be helpful to Celia.

Barbara approaches attorney Morgan to ask for representation in a landlord/tenant suit. Morgan has not handled this type of case before. She is willing to take it on and will charge a much lower hourly rate than her normal rate, but only if Barbara agrees not to sue her for any mistake she might make. Morgan has accordingly drafted a retainer agreement with Barbara that Barbara waives any potential claims for malpractice against Morgan. Morgan has read Barbara this provision and explained it to her, and she has encouraged her orally to seek the advice of another lawyer about whether to hire Morgan on these terms. Barbara says that she fully understands the provision, that she does not need to consult another lawyer, and that she wants to sign the retainer agreement. May Morgan represent Barbara pursuant to this agreement? A. Yes, because Morgan obtained Barbara's informed consent. B. Yes, because Morgan advised Barbara of the desirability to seek the advice of an independent lawyer. C. No, because Morgan did not advise Barbara in writing of the desirability to seek the advice of an independent lawyer. D. No, because Barbara was not independently represented in making this agreement.

D. No, because Barbara was not independently represented in making this agreement.

Attorney Betsy has more clients than she can handle and wants to expand her law firm. She would like to hire two associates but does not have enough capital to do so. Banks will not loan her enough money to pay the associates for the year or so until their fees would cover their own costs. But Betsy's father Roberto, who is a successful investment banker and not a lawyer, would like to provide Betsy with one million dollars to help her expand her business. In return, Roberto would become a limited partner of Betsy, entitled to receive two percent of all of the gross profits of the firm for as long as it exists. Betsy would like to enter into this agreement with Roberto. She and Roberto agree that Roberto will never try to influence Betsy's judgment in rendering legal services. May Betsy enter into this agreement? A. Yes, because Roberto is a member of Betsy's nuclear family. B. Yes, because Betsy will not permit Roberto to influence her professional judgment in providing legal services. C. No, because Roberto will be only a limited partner, rather than a general partner, and therefore not fully responsible for any liabilities of the partnership. D. No, because Betsy may not enter into this arrangement even though Roberto would never try to influence Betsy's professional judgment.

D. No, because Betsy may not enter into this arrangement even though Roberto would never try to influence Betsy's professional judgment.

Jill, an attorney, brings a lawsuit on behalf of her client, Ann, against Grant, after Grant fails to repay Ann's loan to him of $5,000. The retainer provides that Jill's contingent fee will be 30 percent of any recovery. Jill files the complaint one day too late, and the suit is dismissed with prejudice. Jill is chagrinned and embarrassed and wants to make amends for her mistake. She would like to pay Ann out of her own pocket without disclosing her error to Ann or to anyone else. She would simply get Ann a cashier's check for the portion of $5,000 that would have been paid to her if the court had ordered Grant to repay the $5.000. May she do so if she avoids making any false statement about the source of the funds? A. Yes, because this would give Ann the amount she would have expected to obtain as a result of the lawsuit. B. Yes, because she would be doing the right thing without engaging in dishonesty. fraud, deceit, or misrepresentation. C. No, because she is required to report her misconduct to the disciplinary authorities. D. No, because Jill is required to tell Ann that she missed the deadline and that the case was dismissed.

D. No, because Jill is required to tell Ann that she missed the deadline and that the case was dismissed.

Johann, a lawyer, is has recently taken Carly's products liability case. They have agreed to a fee of $250/hour. Because he has only handled a few such cases, he wants to avoid being sued for malpractice. He would like Carly to sign a retainer agreement that provides that any malpractice claim that she wants to make against him has to be resolved by an arbitration rather than in a court. He drafts the retainer agreement and explains it to her but does not tell her that she may have a different lawyer advise her about the desirability of agreeing to the arbitration. Arbitration agreements between professional persons and their patients or clients are not prohibited by state law. She signs the agreement. Is Johann subject to discipline? A. Yes, because Carly was not independently represented in signing the agreement. B. Yes, because Johann did not advise Carly of the desirability of seeking independent counsel before she signed the agreement. C. No, because the matter is not a contingent fee case. D. No, because Johann explained the effect of the arbitration term to Carly.

D. No, because Johann explained the effect of the arbitration term to Carly.

Abdul, a lawyer, is employed by the United States Department of Labor and works in its Office of Civil Rights. He is also an experienced litigator. His neighbor, Blaine, has been having a dispute with the Internal Revenue Service, which claims that Blaine's deduction for home office expenses is not valid and has withheld part of his claimed tax refund. Abdul wants to represent Blaine in a suit against the United States in the federal Tax Court to try to obtain the withheld portion of Blaine's refund. He would not charge Blaine any fee. Also, Abdul has spoken to his supervisor in the Department of Labor, who has confirmed that the Department would have no objection to Abdul providing legal assistance to Blaine in his dispute with the Internal Revenue Service and will confirm this in writing. As a result, Abdul is confident that there is no conflict of interest, and he does not intend to advise Blaine that he should get a different lawyer, because any other lawyer would charge a substantial fee to Blaine. May Abdul represent Blaine in this litigation? A. Yes, because he is not going to charge a fee. B. Yes, because he is obtaining written approval from the Department of Labor. C. No, because he does not plan to advise Blaine about the possibility of obtaining a different lawyer. D. No, because a federal employee may not represent an unrelated client in a claim against the United States.

D. No, because a federal employee may not represent an unrelated client in a claim against the United States.

Harriet intends to bring an action in small claims court against her landlord. She meets with lawyer Joaquin, who is experienced in landlord-tenant law, to see if he will represent her in the matter. After discussing the case, Joaquin and Harriet agree to a flat fee of $1000 for the representation, a reasonable fee given the time and effort Joaquin will expend on the case. Joaquin knows that Angela, a landlord-tenant lawyer three blocks away, would do the same work for $500. In fact, Joaquin believes that Harriets case is so strong that she could probably represent herself and win. Joaquin does not inform Harriet of these things, and he accepts the case. Is Joaquin subject to discipline? A. Yes, because Joaquin was required to tell Harriet that she could resolve her problem without the use of Joaquin's services. B. Yes, because Joaquin was required to tell Harriet that she could achieve the same result while paying a lower fee. C. Yes, because Joaquin must make disclosures to the extent reasonably necessary to permit Harriet to make informed decisions regarding the representation. D. No, because attorneys are not required to tell prospective clients that they could resolve their problems at a lower cost or no cost.

D. No, because attorneys are not required to tell prospective clients that they could resolve their problems at a lower cost or no cost.

Attorney Mosi is representing Blake, a naturopathic doctor accused of malpractice. Blake is accused of having prescribed Amaronset, a homeopathic compound to Eloise, a patient suffering from migraine headaches. Amaronset is not regulated by the FDA and it has not been widely used to treat migraines. Blake did not tell Eloise that the use of this substance was experimental. Three years after her treatment ended, Eloise died from a brain tumor. Her family is suing Blake, who is no longer prescribing Amaronset. A study recently published in Germany found that 1 in 20 of the people who took Amaronset for more than two years developed malignant tumors. During the course of his representation, Blake tells Mosi that he prescribed Amaronset to two other individuals who suffered from migraines. They both took this compound for three years, and in both cases stopped taking it four years ago. These two patients do not know about the recent research on Amaronset or that Eloise died from a brain tumor. Blake refuses to inform these patients about the research or the demise of Eloise, because he does not want to open himself up to further liability. Mosi reasonably believes that it is not reasonably certain that Blakes two other patients are at risk of death or substantial bodily harm. Does Mosi have the discretion to reveal this information to Blake's other patients? A. Yes, because failure to do so would involve dishonesty, fraud, deceit, of misrepresentation. B. Yes, because Mosi may disclose to avoid assisting a criminal or fraudulent act by Blake C. No, because Mosi is required to disclose the information to Blakes other affected patients. D. No, because even if the two patients might have tumors, Mosi must keep this information confidential.

D. No, because even if the two patients might have tumors, Mosi must keep this information confidential.

Lawyer Shawn has handled various property and business transactions for Evelyn, her adult daughter Adelaide, and Evelyn's business partner, Oliver, during the last decade. Those matters are all completed. Five years ago, Shawn wrote a will for Evelyn in which she left all of her property to Adelaide. In that work, Shawn represented and gave advice to both Evelyn and Adelaide. Adelaide, who lives with her mother, suffers from disabling epilepsy and has not been able to support herself. After Shawn finished drafting the will, he drafted a letter to Evelyn and Adelaide informing them that his office was closing their file and confirming the termination of their lawyer-client relationship. However, he failed to mail them the letter. Last week, Evelyn came to Shawn to tell him that it's time for Adelaide to stand on her own two feet. She mentioned that Adelaide has become involved with a boyfriend, Felix, and has been staying out nights. She would like Shawn to write a new will for her in which she would leave all of her property to Oliver. She further asks Shawn not to reveal to Adelaide the contents of the new will. May Shawn write the new will for Evelyn? A. Yes, because his prior representation of Adelaide was incidental to his representation of Evelyn. B. Yes, because he will not reveal the contents of the new will to Adelaide without authorization by Evelyn. C. No, because he failed to inform Adelaide that he no longer represents her. D. No, because he did not obtain Adelaide's informed consent to his representation of Evelyn with respect to the new will.

D. No, because he did not obtain Adelaide's informed consent to his representation of Evelyn with respect to the new will.

Attorney Mort represents Julio, who was injured in an automobile accident caused by Yvonne. Julio has back pain and frequent, severe headaches. Julio is suing Yvonne and will soon have his deposition taken. When Mort interviewed Julio and asked him how frequently he had severe headaches, Julio said, "I don't have them every day, and sometimes they aren't so bad. But about four days a week, I wake up with a severe headache.? Mort told Julio that minimizing the frequency of his head pain would weaken his case. Julio asked Mort what he should say. Mort said, "Well you don't have to say, unless you are asked, that you don't have headaches every day, or that they aren't so bad? Julio said, "So, what should I say?" Mort said, "I understand that you wake up most mornings with severe headaches? Isn't that right?" Julio said, "That's right." Mort said, "When I ask you about your headaches, you can say: 'Most mornings, I wake up with severe headaches: Julio testified in the deposition that he woke up most days with severe headaches, and he was not asked more questions about this subject. The case was settled shortly thereafter. However, Julio got into a dispute with Mort about the calculations of Mort's fee, and Julio filed a bar complaint against Mort. During the investigation by the bar counsel's office, Julio described how Mort had coached him to testify regarding his headaches. Is Mort subject to discipline for his conduct regarding Julio's testimony? A. Yes, because he advised Julio not to use words that would minimize the frequency of his head pain. B. Yes, because he told Julio exactly what to say, which was different from what Julio would have said if he had not been coached. C. No, because Mort's coaching was only in connection with a deposition, not a trial D. No, because he did not tell Julio to lie.

D. No, because he did not tell Julio to lie.

Maude, a lawyer, was retained by Anya, who lives in an assisted-living building for elderly persons. Anya noticed that many residents of the building were contracting respiratory infections. Anya has noticed that the ceiling tiles by some of the air vents in her apartment have turned black, an indicator of mold in the HVAC system. She suspected that Warner Management, Inc., which was running the building, was cutting costs, and had not been servicing the air conditioning and heating systems often enough. Regular servicing and changing the filters quarterly is necessary to maintain clean indoor air Maude wants to question Xavier, the Warner employee who services the heating and air conditioning systems and changes the filters at Anya's building. Under the substantive law of the jurisdiction, if Xavier reveals that he failed to service the system or change the filters regularly, Warner Management could be liable for his negligence on the theory of respondeat superior. Maude knows that Warner is represented by attorney Peter in all matters related to Anya's building. Maude is planning to tell Xavier that she represents Anya but is not planning to tell him that he has a right to consult with counsel of his choice. Also, she does not plan to ask Peter's permission to question Xavier, or even to notify Peter that she is interviewing him. May she question Xavier without getting Peter's permission? A. Yes, because a lawyer may always interview the employee of a corporation that is an adversary of her client. B. Yes, because Xavier is not an officer or director of Warner or a member of Warner's management group. C. No, because while Anya should notify Peter before interviewing Xavier, she does not need his permission to conduct the interview. D. No, because his failure to change the filters often enough could be imputed to the organization for the purpose of civil liability.

D. No, because his failure to change the filters often enough could be imputed to the organization for the purpose of civil liability.

Hussain, a lawyer, represents Surety, Inc., an automobile insurance company that insures rental car companies. Lee was seriously injured when the brakes on his rental car failed. Through his lawyer, Esther, Lee sued the rental car company, and pursuant to his agreement with Surety, Hussain represented the defendant in the lawsuit. Surety's investigation revealed that the brakes were faulty and the rental car company is therefore liable. Hussain knew that Esther had not done her own forensic investigation of the brakes. Surety authorized Hussain to offer Lee up to $500,000 to settle the suit. When Esther and Hussain met to explore whether a negotiated settlement was possible, Esther said her client was willing to accept a settlement of $400,000. Hussain said, "I know that my client won't pay a penny more than $300,000' Is Hussain subject to discipline for lying to Esther? A. Yes, because a lawyer may not make a false statement of material fact to a third person. B. Yes, because lawyers may not make false statements about how much a client is willing to offer in settlement. C. No, because lawyers are permitted to make false statements to other lawyers, provided that the statements are not made under oath or in the course of a proceeding. D. No, because lawyers are allowed to make false statements about a client's intentions regarding an acceptable settlement.

D. No, because lawyers are allowed to make false statements about a client's intentions regarding an acceptable settlement.

Duncan, an attorney, would like to meet the pro bono standard in the rules. However, he is extremely busy and barely earning enough in fees from his paying clients to pay his bills and feed his family. Some of his clients earn only the minimum wage and their family incomes put them below the federal poverty level. If all Duncan does to provide pro bono services is to devote 40 hours of legal services per year to such clients at a 50 percent discount from his usual fee, would he be subject to discipline? A. Yes, because he should provide most of his hours of "pro bono" service without charging any fee. B. Yes, because clients who are employed at the minimum wage are not indigent. C. No, because despite earning barely enough to pay his bills, he discounts his bills for clients whose family incomes are below the poverty level. D. No, because lawyers are not disciplined for providing few pro bono services, or even no such services.

D. No, because lawyers are not disciplined for providing few pro bono services, or even no such services.

Attorney Lilith represents Opal in a custody case against Opal's ex-husband, Claude. Lilith knows that Claude is represented by another lawyer, Oscar. Claude enters Lilith's office one day and tells her that he wants to speak with her about Opal's treatment of their children. Lilith says, "As you know, Oscar represents you in this matter." Claude says, "I know that, but I want to talk to you anyway." May Lilith allow Claude to continue? A. Yes, because she did not initiate the conversation; Claude did. B. Yes, because she has obtained Claude's oral consent after reminding him that he was represented by Oscar. C. No, because she did not obtain Claude's written consent. D. No, because she does not have consent from Claude's lawyer.

D. No, because she does not have consent from Claude's lawyer.

Attorney Ria represents criminal defendant Carl, who is charged with armed robbery. Carl is in jail pending his trial. During a meeting at the prison, Carl tells Ria that on the night of the alleged robbery, he was at a hockey game with his girlfriend. When Ria returns to her office, she does an Internet search and finds out that the hockey game actually took place on the night before the robbery, not the night of the robbery. Ria tells Carl this information during their next meeting, and Carl tells her he was mistaken. He says he went to the hockey game the night before the robbery, and on the night of the robbery, he was having dinner with his mother an hour away from where the robbery took place. Ria reasonably thinks that Carl probably is lying about the dinner with his mother. Without first counseling Carl to tell the truth, she refuses to allow Carl to testify about the dinner at his trial. Is her refusal proper? A. Yes, because her belief that Carl is lying is reasonable. B. Yes, because she believes that Carl is lying, and the reasonableness of her belief is not relevant. C. No, because she was required first to counsel Carl that he should tell the truth in court, and she could only refuse to allow him to testify that he was at dinner with his mother if he persisted in doing so after this caution. D. No, because she does not know for sure that Carl is lying.

D. No, because she does not know for sure that Carl is lying.

Athena practices law in a city of medium size. She specializes in trusts and estates and in elder law. For many years, Athena lived next door to Meredith, an elderly widow. Meredith lived with her middle-aged daughter Dora, but spent most of her time with her boyfriend, Sven. Athena did not do any legal work for Meredith, though she got to know her well. At Meredith's request, a year ago Athena acted as one of two subscribing witnesses to Meredith's most recent will. This new will replaced an earlier will which had left everything to Dora. The new will left $500,000 to Dora, $1,000,000 to Sven, and the rest of Merediths multi-million-dollar estate to charity. Meredith named her cousin Ernest as her executor. The other witness to the new will died in February. Meredith died last week. Dora has announced that she plans to contest the will on the ground that Meredith was incompetent when she executed the will. Athena probably will be required to testify in the probate proceeding on the subject of Merediths competency at the time she signed the will. Ernest, the executor, has asked Athena to represent him and defend the will in the probate proceeding. Athena has never before represented Dora, Sven, or Ernest. Athena wants to represent Ernest in probating the will, and also to testify truthfully that she knew Meredith well and that Meredith appeared to her to be competent. Is her proposed conduct proper? A. Yes, because she plans to tell the truth. B. Yes, because she does not represent and has never represented any of the heirs. C. Yes, because her testimony would support her client's position, so she would not have a conflict with her own client. D. No, because she is a necessary witness on a contested issue.

D. No, because she is a necessary witness on a contested issue.

Erin, a lawyer, had a motion that was scheduled to be argued on November 5. Another attorney, Manfred, represented her adversary. On October 31, Erin learned that her son's surgery had been scheduled for November 5, and she wanted to be with her son on the day of the surgery. She tried to call Manfred to discuss rescheduling, but he was out of town and could not be reached that day. So without first notifying Manfred, she telephoned the judge's clerk to find out whether the argument on the motion could be heard the following week. To her surprise, the judge picked up the phone and explained that he had answered because his clerk was at lunch. Erin explained the situation and asked the judge whether the argument could be scheduled for November 12. The judge checked his computer and advised that he could fit in the argument on that date, and he advised Erin to notify Mantred of her request and if Manfred had no objection, to telephone his clerk in a few hours. He said that his clerk would also send a notice to her and to Manfred, describing his conversation with Erin and notifying that in the absence of any objection, the hearing would be rescheduled as requested. Is Erin subject to discipline? A. Yes, because she didn't notify Manfred that she was going to telephone the judges chambers about rescheduling the hearing. B. Yes, because she spoke directly to the judge rather than to his clerk. C. No, because it was not her fault that the judge picked up the telephone. D. No, because she only discussed a procedural issue with the judge.

D. No, because she only discussed a procedural issue with the judge.

Chris and Bobby are accused of burglarizing a house. They want Meyer to represent both of them in the criminal matter because they prefer not to have to pay two lawyers. Also, they believe that if they coordinate their stories and refuse to cooperate with the police, the state won't have enough evidence to convict either of them. No state statute prohibits lawyers from representing criminal co-defendants. Meyer obtains the police reports and charging documents. He also obtains the criminal records of both Chris and Bobby. From his investigation, Meyer learns that Chris may have been the instigator. Chris is 26 years old and Bobby is 19. Chris has three prior felony convictions, while Bobby has a prior conviction for possession of a small amount of marijuana, for which he received a suspended sentence that could be revoked if he is again convicted. Meyer's law clerk recommends that Meyer obtain a court order permitting the joint representation, but Meyer declines to seek such an order because he believes that he can provide competent and diligent representation to both defendants. May Meyer represent both defendants if each gives informed consent to the joint representation, confirmed in writing? A. Yes, because both clients have provided consent confirmed in writing. B. Yes, because the charges arise out of a single incident. C. No, because Meyer did not seek an order from the judge allowing the joint representation. D. No, because the joint representation presents a non-consentable conflict.

D. No, because the joint representation presents a non-consentable conflict.

Client Sylvia hired lawyer Dino to represent her in a personal injury action against Chainmart. In the course of the representation, Sylvia told Dino that she was prepared for the case to go to trial, if necessary, and that she probably would not even consider accepting a settlement offer below $2 million. Dino thought she might be awarded $10 million or more if the claim were decided by a jury. The evening before trial, opposing counsel called Dino and offered a settlement of $1 million. Dino tried calling Sylvia but could not reach her. Five hours after trying unsuccessfully to reach Sylvia and hoping to be able to get some sleep before the trial, Dino called opposing counsel and rejected the offer. The case went to trial the next morning, and after trial, the jury awarded Sylvia $5 million. Is Dino subject to malpractice liability for his actions? A. Yes, because decisions to accept or reject settlement offers are to be made by the client. B. Yes, because a lawyer has a duty to keep his client informed of all settlement offers. C. No, because Sylvia authorized Dino to reject any offer under $2 million. D. No, because the jury award was greater than the settlement offer.

D. No, because the jury award was greater than the settlement offer.

Samir practices family law. His clients often have problems that might better be addressed by a marital counselor, who might be able to help a divorcing couple to reconcile or to separate more amicably. He wants to form a partnership with his cousin Veena, a licensed social worker. They would share a suite of offices, and a client could receive services from either or both of them, as the client preferred. Samir and Veena want to share the fees earned by either of them, in proportion to the work done by each. Samir will take steps to ensure that Veena complies with the Rules of Professional Conduct. Veena will not direct or regulate Samir's professional judgment in rendering legal services, and all clients will be informed that only Samir is licensed to practice law. However, Samir does not plan to seek each client's written consent to receive legal services from a partnership that includes a social worker who is not also a lawyer. May they create this partnership? A. Yes, because Samir will take steps to ensure that Veena complies with the Rules of Professional Conduct. B. Yes, because the fees will be shared in proportion to the work done by each. C. No, because Samir does not plan to seek written consent from his clients to be served by a partnership that includes a non-lawyer. D. No, because the partnership would be prohibited in any event.

D. No, because the partnership would be prohibited in any event.


Related study sets

1535 Nutrition, Health Promotion, Teaching

View Set

Умножение и деление натуральных чисел 5 класс

View Set